Sie sind auf Seite 1von 67

Federal Income Tax Professor Morrison Fall 2003 CHAPTER 1: Introduction

A. The Constitution and the Income Tax


A. Tax Payers and Tax Rates 1. Progressive Rate System: Income-tax rates for individuals and corps have almost always been progressivei.e., tax on a high income is a larger percentage of the income than the tax on a lower income. i) Proportional (not used): A tax on the same percentage of all income, i.e., 20% of all income. (A-100K, B-20K, after tax A would have 80K, while B would have 16K, A still has 5X the amount of income as B) ii) Regressive (not used): A tax on the larger percentage of lower income than of higher incomes, i.e., a tax of 20% of the first 100K, plus 10% of the amount of income over 100K. *After the progressive rate system is implemented, as a general rule, if As before tax income is higher than Bs before tax income, As after tax income will also be higher than Bs. *A variety of tax provisions subject various kinds of income to lighter than normal taxation, to further non-tax policies favored by legislators B. Tax-Rate System: The tax-rate schedules of 1 apply to the taxpayers taxable income. Taxable income is computed by subtracting from the taxpayers gross income (See 61) any allowable deductions. 61 Gross Income Definedencompasses only the gross amount of wages, dividends, interest, rent, etc received by the taxpayer undiminished by any deductions attributable to such income HYPO: (Progressive Structure) A, single male, has $100K of taxable income. Rate scheduleOver $53,500 but not over $115, 000is taxed at $11,493, plus 30% of the excess over $53,500. $11,493 is automatic With 100K, A is $46,500 over $53,500. This amount is taxed at 30% (46,500 x .30) and equals $13,950 of additional tax. Tax Liability of A: $25,443 ($11,493 + $13,950) As marginal tax rate is 30% As average tax rate is 25.433% (divided by 100K) HYPO: A earns $50K a year, for ten years. B earns nothing in yrs. 1, 3, 5, 7 and 9 and 100K in yrs 2, 4, 6, 8. Rate Schedule0% of the first $25K a year of income plus 10% of the amount by which the taxpayers income exceeds $25K A, each year is taxed $2500. $25K for ten years B, is taxed nothing in 1,3,5,7,9. But in years 2,4,6,8,10 is taxed at a rate of 10% on the $75K. $7500 per year, or $37,500K total for years 2,4,6,8,10 C. Internal Revenue Service *Major function is to see that taxes are collected *Letter rulings in response to taxpayer requests for rulings on contemplated transactions are made public, but they are not meant to have precedential value. *A ruling request must contain a complete statement of the facts relating to the transaction in question and copies of relevant documents.

D. Tax Controversies: Taxpayers are required to file returns and pay the taxes due. If there is a controversy w/ reportable income, the following procedure is implemented: (1) Action by the IRS: Most are settled by agreement b/t the taxpayer and Revenue Agent. If not, the IRS will send a 30-day letter explaining its determination/position. If no action is taken w/in 30 days by the taxpayer (a written protest or payment), a deficiency notice, or 90-day letter will be sent. Taxpayer now has 90 days to pay up or file a petition w/ tax court. If neither is done, the IRS will assess the tax and demand payment by certain date, and then seize assets if that order is not complied with. (2) Action by the Taxpayer: Possible avenues for the taxpayer in a dispute. Taxpayer should, depending on sum of the dispute, do research to determine the best possible course of action and/or venue. (a) Contest liability in tax court (cannot be availed if taxpayer has paid the tax in full, b/c tax court is Art. I court, only having deficiency jxd) No jury, tax-court judge. Appeal would go to the circuit court of appeals, then to SC, which rarely grants cert. in tax cases. (b) If you have money, pay it, then sue for refund in the federal Refund Cases district court. Probably good if you are weak on the Code. You (b) & (c). Gov is will get jury. rep. by the DOJ (c) Pay money, sue for a refund in court of fed. claims, appeal to the federal circuit in D.C. *Taxpayer has a free choice of forum in tax cases, and b/c tax decisions of courts other than the SC are not always binding on other courts, questions of tax law often remain unsettled from many years. COA decision binds DC in that circuit Court of fed. claims must follow decisions of the COA for the Fed. Cir. Tax Court, must follow (per Golsen Rule) decisions of the court of appeals to which the case before it is appealable. Tax Court is an Article I court, which only has deficiency jurisdiction, and cannot be availed by a taxpayer who has paid the tax in full. (3) Statute of Limitations: Time in which IRS must come after you and assess tax (a) 3 Years: S.O.L. from the date on which the return was filed (b) 6 Years: If return omits an item of gross income greater than 25% of the gross income shown on the return (c) NO S.O.L: No Return filed, or files on in which that is false or fraudulent, with intent to evade tax. CHAPTER II: The Concept of Income A. Basic Tax Computations *Taxable Income: Calculated by subtracting the taxpayers deductions from gross income. (If gross income is 200K, and deductions are 50K, taxable income equals 150K) (1) Individual Taxpayers: Calculations of taxable income involves two steps (a) Deductions (from 62above-the-line deductions) are subtracted from gross income, yielding a figure called adjusted gross income (AGI) *62 encompasses all business expenses (except most employee business expenses), investment expenses pertaining to rents. (b) Next, the taxpayer subtracts deductions for personal exemptions (not tied to actual outlays) and either (i) itemized deductions or (ii) standard deduction from AGI to

determine taxable income. (If your itemized exceeds your standard, you deduct the itemized. If your itemized is less than the standard, you deduct the standard) *All taxpayers will deduct their above-the-line deductions and their personal exemptions. *Itemized deductions: deductions other than above-the-line deductions, personal exemptions and the standard deductions, i.e., charitable contributions, home mortgage interest, state and local income taxes, property taxes *Standard deductions: Statutory Amount (usually 5K for married couple, 3K for individual) (2) Child Tax Credits: Low and middle income parents are allowed, as well as their personal exemptions, credits for their US citizen children under the age of seventeen. $600 for each qualifying child Phased out if income exceeds $110K (married filing joint return) or $75K (individual). For each 1K over the statutory amount, the total credit is reduced by 50$. Hypo: 2 qualifying kids, income is less than $110K, $1200 credit If their income is 115,200, their credit is reduced by $300, to $900. HYPO: Married couple w/ two dependent children has $60K gross income and the following deductions: (1) $2K loss on the sale of business property (2) $8K mortgage interest on principal residence (3) $1K charitable contribution (4) $4K state and local income taxes (5) $2K property taxes on principal residence *(1) is the only above the line deduction ( 62(a)(1) & (3), which totals 2K, therefore their AGI is $58K. They will next subtract itemized deductions and personal exemptions *(2)(5) are itemized deductions totaling $15K. (58-15=45) *Personal deductions of 2K each for themselves and kids, totaling $8K. (45-8=35) TAXABLE INCOME: $35K. *Rates: $1,200 plus 15% in excess of $12K. Excess $23K at 10% equals $3450 TAX LIABILITY: $4650 ($1200 + $3450) From this amount the couple subtracts their $960 credit for child-care expenses and a $1200 child tax credit, yielding a tax of $2490 (An above the line deduction will be worth more to many taxpayers than an identical itemized deduction would be worth. b/c only those whom total itemized deduction exceed their standard deductions will claim itemized deductions on their returns. You always claim above the line deductions). HYPO (page 42): A, age 67, has 40K gross income, 4K of deductions from rental property, 1K of property taxes on her home, and $2650 of state income taxes. AGI: { 62} $36K (Above the line deductions-$4K subtracted from gross income-$40K) i) Itemized Deductions: $3650 ii) Standardized Deductions: $5750 ($3K + $2K + $750) Standard Deduction of $3000 (see 63(c)(2)(C)) for an individual who is not married and who is not a surviving spouse or head of household. Over 65 add-on of $750 (see 63(f)(3) this is only for standard deduction) for person whom is not married and is not a surviving spouse. You get $750 instead of $600. Personal Exemption: $2000 (see 151(d)(1)) Taxable Income: $30250 (Gross income, $36K, minus deductions allowed by 63, $5750.) Taxable Income defined in 63.

(3) Wealthy Taxpayer: Taxpayers w/ higher incomes face two additional complications, both of which were introduced in 1990 to raise revenues. Both of these limitations are to be phased out beginning in 2006: (A) Phase-Outs: 151(d) phases out the personal exemption for taxpayers whose AGI exceeds threshold amounts. The personal exemption deduction for a taxpayer w/ AGI over the threshold amount is reduced by 2% for each $2500 of AGI over the threshold. i) Threshold Amounts: $150K for joint returns or surviving spouse $100K for individual not married not a surviving spouse $75K for married individual filing separate return. HYPO: Married couple filing joint return has AGI of $250K, their AGI exceeds their threshold by $100K. Their personal exemptions are therefore reduced by 80%. (The 100K by which AGI exceeds the threshold is 40 times $2500, and the reduction is two percent for each of those 40.) 100K / 2500= 40 x 2=80 151(d) provides that in no instance may the percentage exceed 100% (B) Reduction of itemized deductions: 68 limits the deductibility of itemized deductions by taxpayers having AGI in excess of a $100K threshold (50K for married taxpayers filing separate returns) *Generally, the total amount of the taxpayers otherwise deductible itemized deductions is reduced by 3% of the excess of AGI over the threshold. However, the reduction cannot be more than 80% of total itemized deductions. i) Exception for certain itemized deductions: 68(c) indicates that for purposes of this section, itemized deductions do not include deductions for medical expenses, casualty losses, investment interest, and wagering losses. ii) Standard v. Itemized: This does not affect those who take the standard deduction rather than itemizing HYPO: A is unmarried taxpayer w/ AGI of $120K, itemized deductions of $20K (before phaseout), and one personal exemption of $2K. AGI exceeds the threshold by $20K, and otherwise deductible itemized deduction must be reduced by 3% of the excess ($20K) Reduction in itemized deductions: $600 Result: This increases the amount of As taxable income by $600, which will add $180 (30% of $600) to her tax bill, as she is in the 30% marginal bracket) If her itemized deductions had been $25K or $15K, instead of $20K, the increase in taxable income would still have been $600. HYPO: H&W, married and file a joint return, have an AGI of $200K for the current year. They are entitled to four dependency exemptions (for their kids) under 151. They have $30K of itemized deductions that are subject to the limits of 68. Six dependency exemptions: $8K (Applicable amount is 150K) -AGI exceeds the threshold by 50K -$50K / $2500 = 20 x 2% = 40% reduction in each personal exemption of $2000, which comes out to be $800. $2000-800 = $1200 -1200 x 6 (# of exemptions) = 7200. -Phased-Out Exemptions: $7200 Itemized deductions: 30K (Applicable amount is 100K) -AGI exceeds threshold by 100K, and otherwise deductible itemized deduction must be reduced by 3% of the excess (100K) or $3,000 (100,000 x .03)

-Phased Out Itemized Deductions: $30,000-$3,000 = $27,000 TAXABLE INCOME: $165,800 (AGI minus Exemptions and Deductions: 200K 27K $7200). B. Accessions to Wealth: What is considered income 1. In General: No single conclusive criterion has been found to determine in all situations what is a sufficient gain to support the imposition of an income tax. Commissioner v. Glenshaw Glass Co. I: Does $ received as punitive damage count as gross income under 61 which must be reported? Glenshaw didnt report $324K punitive portion of settlement as income Taxpayer claimed that income is derived from capital, labor or both. Punitives do not fit w/in this definition. H: Yes. Taxable income. Undeniable accessions to wealth. R: Court gives a liberal construction to this broad phraseology in recognition of the intention of Congress to tax all gains (accessions to wealth) except those specifically exempted. (This is a broad statement that probably shouldnt be taken to an extreme.)Mere fact that payments were extracted from wrongdoers as punishment cannot detract from their character as taxable income to the recipients. Eisner v. Macomber I: Does the distribution of a corporate stock dividend (as opposed to a cash dividend, which is obviously income) constitute a realized gain to the shareholder H: Distributions is not a taxable event, this tax statute violates 16th Amend. R: Taxpayer received nothing out of the companys assets for his separate use and benefit. Ms. Macomber never realized any income. In order to realize, Ms. Macomber would have had to sell. Court describes income as the gain derived from capital, from labor, or from both combined.

i)

Other Windfall Cases: Found money or windfalls (e.g., winning the lottery are taxable) Cesarini v. US: C bought a second hand piano for $15 and seven yrs. later found $4467 in cash hidden in the piano. (Taxpayer argued that it may have been income when I found piano, but SOL has long since run) Relying on Glenshaw, the court held that the found money was income in the year that it was found. T. Reg. 1.6114 provides that treasure trove (property found by the T) is income in the year when it is reduced to undisputed possession. What if C had found a valuable object rather than cash? If C had found it the piano is worth $500K, it is not income until it is realized/sold. $10 basis in a $500K pianogain would be the difference ($499,990) Treas. Reg. 1.61-14(a) has generally been read as requiring inclusion in income of the value of the found property.

161-1(a)Gross Incomemeans all income from whatever source derived, unless excluded by law. Includes income realized in any form, whether money, property, or services. Income, therefore, may be realized in the form of services, meals, accommodations, stock, or other property, as well as in cash. 161-14(a)Misc. items of gross incomeindicates that in addition to 61(a), there are many other kinds of gross income. Illegal gains, treasure trove, punitives, etc

ii)

Realization: Before any increase in net worth becomes taxable, a realization must occur. This means that a crystallizing event must have taken place which makes it reasonable and convenient to compute gain or loss. See Code 1001 (sale or disposition of property)

HYPO: A taxpayer who finds a gold coin worth $100 has realized that amount of income under Treas. Reg. 161-14(a), but a taxpayer who buys a gold coin for $50 has no income when the coin increases in value to $150 (unless he sells or exchanges it for something else). In each case, wealth has increased, but in the second, the gain in question has not been realized HYPO: T owes X $1K. T transfers property worth $1K, but on which Ts basis (cost) was only $400. There is realization of a $600 gain to T. HYPO: T discovers a valuable natural gas below houseYou must have undisputed possession. If you extract and sell, then you have realized b/c you have undisputed possession. HYPO: Tenant erected a bldg. on leased land and then defaulted on lease payment. On regaining possession, the landlord was held to have realized income equal to the value of the bldg. Read: Realization must occur before an increase in net worth becomes taxable. An event must occur that it is appropriate to tax the increase now. 2. Gross Income From Sales a. Appreciation: The tax system usually does not tax appreciation (or allow a deduction for losses) until the taxpayer disposes of the property. (Until the property is disposed of appreciation or depreciation are unrealized) Difficult to make annual appraisals to ascertain how much appreciation has occurred. Thus b/c unrealized appreciation is not taxed, people will prefer investing in assets that will appreciate to those assets that will produce realized gains. To wit: You would prefer to invest in stock that will increase in value, as opposed to the stock that pays increases in dividends and remains the same price. HYPO: L buys land for $100K. One year later, oil is struck next door, and her land shoots up in value to $500K. Oil is never found, but there is that potential. Lucy does not have to pay any tax right away, only when she sells for 500K, 40 yrs. later. HYPO: Taxpayer buys stock for $1000 on June 15, by the end of the year, the stock has increased in value to $1500. Not taxable until property is disposed of. HYPO: If the same taxpayer sells the stock in the next year for $1700, the sale is a disposition (or a realization event) which requires the taxpayer to reckon gain or loss on the transaction. The profit on the transaction is only the $700 excess of the amount received upon the sale ($1700) over the original investment ($1000), i.e., not everything that comes in is income. (At the time of the sale, the gain is realized and usually recognized, but the sale itself did not enrich the taxpayer. The sale consists of an exchange of property worth 1700 for cash equivalent. The enrichment or accession to wealth, took place whenever the stock increased in value from $1000 to $1700. 61(a)(3) [g]ains derived from dealings in property are included in gross income. HYPO: If the same taxpayer had sold the stock for $900, the gross income on the sale would have been zero: No such thing as negative gross income. This taxpayer has realized a loss of $100, and this loss will be dealt w/ by allowing the taxpayer a $100 deduction in calculating taxable income. 1001(a) Computation of gain or loss: A taxpayers realized gain on the disposition of property is the excess of the amount realized over the adjusted basis. In the previous hypos, amount realized is simply the amount of cash for which the taxpayer sells the property, and the propertys basis is its $1000 cost. 1001(b) Amount Realized: The amount realized from the sale or other disposition of property shall be the sum of any money received plus the fair market value of the property received 1001(c) Recognition of gain or loss: Gain or loss realized on the sale or exchange of an asset is recognized (taxed) unless another provision of law provides otherwise.

1012 Basis of PropertyCost: The basis of property shall be the cost of such property. The cost of real property shall not include any amount in respect of real property taxes which are treated under 164(d) as imposed on the taxpayer. (In some instances, treated elsewhere in the code, a propertys basis must be adjusted and basis may be something other than the cost of the property). Basis must be known both for computation of gain or loss and also computing depreciation 1011(a) Adjusted basis for determining gain or loss: The adjusted basis for determining the gain or loss from the sale or other disposition of property, whenever acquired, shall be the basis (see 1012, i.e., cost), adjusted upward for expenditures, receipts, losses, or other items properly chargeable to the capital account, and adjusted down for depreciation, amortization and depletion as provided in 1016 HYPO: A finds and keeps a gold statute worth $10K. If in a later year she sells the statute for 12K, how much gain does she realize upon the sale? $2K gain on the sale (she must pay initial tax on the 10K she realizes) HYPO: B buys a statute from an antique dealer for $4K, actual value is $14K. Later, B sells for $16K. What is Bs gain on the sale? $12K (4K basisno gain until sold). HYPO: Client comes to M for will preparation. Gives M a chicken for preparation, says it is worth $25. $25 worth of income is realized If M sells the chicken to his law partner for $30 M has a gain of $5, and is taxed only on that gain HYPO: M buys a car for 20K, W buys the same car of 17K. Wife does not have income. Just bargaining b/t parties. HYPO: W, as the 10,000 buyer gets a check for $10K from the dealership. Windfall, not related to the transaction. INCOME. HYPO: M renders services to widow, and never charges her. M, with intention of moving to St. Louis, meets w/ widow, whom gives him $300. As no payment for services was ever discussed, M will argue that it is a gift or windfall, not tied to any particular service rendered. The IRS will say compensation for services. HYPO: C receives a $500 Fed. income-tax refund. Not includable in gross income.

b. Realization and Recognition: A realized gain is includable in gross income only if it is also
recognized. Except as provided in the Code, the entire amount of any realized gain or loss on the sale or exchange of property is recognized for tax purposes. (Read: If not recognized, not taxable) Powerful presumption of the Code is that all gains or losses realized shall be recognized Congress has provided for non-recognition of gain (or loss) in some situations 1001(c). The rationale being that a taxpayer who has engaged in this class of transactions has not changed the nature of his investment sufficiently to warrant a tax being imposes on his realized gain. An example of a non-recognition provision is 1031, under which gain from the exchange of business/investment property for other business or investment property of like kind is not recognized), corporate mergers.

c. Bargain Purchases; Employee Discounts: When a taxpayer purchases property at arms length
for less than it is worth, he is ordinarily not taxed on the fruits of his astute dealing. Since the propertys basis is its cost, however, the profit on the advantageous purchase may enter into gain on a later sale of the property. In contrast, the bargain element may be taxed immediately if a bargain purchase for less than FMV is not an independent arms length transaction, but reflects an extraneous objective, such as the sellers desire to confer an economic advantage on the buyer. HYPO: House is for sale: $75K. House is appraised at $100K. Not income per 61 to the buyer. HYPO: M gives S $10K after bad legal advice which cost her $10K. Not income to S. S is just being returned to original position. (This payment served only to restore Ss impaired capitalsee Tax Free Recovery of Capital) 3. Compensation for Services/Payment of Taxpayers Expenses Old Colony Trust Co. v. Commissioner F: Wood, President of American Woolen, was paid salary and commission of $1M in 1918. Company agreed to pay Woods fed. income tax of $680K. (Amendment goal was to make sure that tax paid was at the highest progressive rates). I: Did the payment by the employer of the income taxes assessable against Wood constitute additional taxable income to Wood? H: Payment by employer constituted income to employee, and Wood (employee) is taxable on the $680K. R: Voluntary payment was irrelevant, and this was not a gift. Woods debt was discharged, making him wealthier. The transaction was in the nature of compensation for services, which is explicitly taxable under 61(a)(1) Woods argument that the $ was never paid to him, it was paid to the IRS directly, was rejected. (Still income although you dont receive cash in hand) *OLD COLONY PRINCIPLES: (1) Form of Income does not matter (2) Income may include discharge of obligation to 3rd party (As payment of Bs obligation is tantamount to a payment from A to B) (3) In employment context, gift argument is last-ditch effort, that probably wont work.

a. Tax upon a tax: Pursuant to Old Colony, this type of employment


agreement is taxable. Therefore, consider the following: If the tax rate is 40% and employee T is paid $30K, and like Old Colony, the employer pays the 12K tax. This is income to T (Old Colony), and another tax is imposed upon this income. The employer would then pay $4800 more tax on behalf of T. $4800 is then income to the T, etc HYPO: C transfers property w/ an adjusted basis of $40 and a value of $100 to D in exchange for $70 cash and Ds payment of Cs $30 debt to E. What result to C and D? C sold property, $60 taxable gain D basis of $100 HYPO: E is employed by X corp in a state which has no income tax. Last year, E accepted a temp. assignment for X in an adjoining state which imposes a 5% income-tax on compensation earned w/in the

state. E agreed to serve at his regular salary if X would pay any state income tax liability incurred by E at the temporary post. E earned $10K while on assignment. X paid the resulting $500K tax liability. Must E include the $500 in Fed. gross income? Does it matter whether the $500 is included? Consider 164(a) (3) ASK MORRISONHe says income

b) Payments by Liability Insurers: When damages resulting from a


taxpayers tortious behavior are paid by an insurance company, as in the case of an automobile accident caused by the taxpayers negligence, the payment is not taxed as income to the T. This is probably a corollary of the denial of a loss deduction for premiums paid by an insured who sustains no reimbursable loss. HYPO: M owes $1M in damages to a neurosurgeon whom he hit. Hartford pays Ms obligation. Is this income to M? NO.

c) Obligation of Payor: In Old Colony, the employee was the beneficiary


of the employers payments (since he would have been required to pay the taxes from his own resources if the Co. had not.) However, Old Colony does not apply to payments satisfying obligations of the payor, e/t the employee derives some benefit as a by-product of the payment (e.g., corporate obligationimposed by state lawto pay s/h legal fees in derivative action; employers payment of business expenses charged to an employees account) i) Transfer of property as Compensation for Services: 83 Commissioner v. LoBue F: LoBue was a manager of Michigan Chemical Corp. Co. had a stock option plan making 10K shares of its common stock available for distribution to key employees at $5 per share. Ultimately, LoBue exercised these $5 per share options, paying the company only $1700 for stock having a market value of $9,930. ($29/share). LoBue gave promissory notes when stock was $20/share (83if you pay for stock w/note (secured by the stock itself) and no personal obligation, it is really still just an option.) Tax Court held for LoBue, finding options not to be compensation, but a proprietary interest in the business. H: Revd. This is compensation for personal service w/in meaning of 61. R: 1) This is not a gift, so it seems impossible to say that it is not compensation. No exemption exists which allows deductions for proprietary interests. When assets are transferred by an employer to an employee to secure better service they are plainly compensation, regardless if it is stock or money. 2) LoBue realized taxable gain when he purchased the stock 3) When should the gain on the shares be measured? (This is remanded to the tax court) When Lobue paid the cash and the market value was higher or when the options were granted or when the promissory notes were given and the market value of the shares was lower? $5 (Option-granted)No tax is payable on the option $20 (x340 shares)Did he have a real obligation? Basis would be $20. If he sells, the difference is capital gain $29 (340 shares)Lobue would not want to be taxed here, as it would be the most costly. IRC 83Property transferred in connection with the performance of services (this does not only apply to employees, but also a lawyer who gets stock) Basic Rule(s): If you buy stock from employer, for less than it is worth, the spread is income, if there are no restrictions.

OptionNo income at that point. It becomes so when you exercise the option, and there is no restrictions (substantial risk of forfeiture) BasisDollars spent and amount taxed. 83 deals with:

1) Non-Qualified Stock Options: Stock transferred to an employee for a bargain price


and subject to restrictions (Not transferable, subject to forfeiture). *If the stock is transferred for a bargain price and not subject to restrictions, the result would be immediate compensation income of the bargain element. a) Restrictions: If there are restrictions, then the employee does not have to include the bargain element of the transaction in his income until the substantial risks of forfeiture are lifted. *Not very marketable *When restrictions are lifted, the employee includes as ordinary income the value of the stock at the time the restrictions are liftednot the value at the time of the transfer. *This amount is reduced by the employees purchase price.

2) Deferred Compensation Arrangementsdefer receipt of some of my income to


when marginal rates are (hopefully) lower. 1.83-1: Generally such property is not taxable until it has been transferred (as defined in 1.83-3(a)) and becomes substantially vested (as defined in 1.83-3(b)). The fair market value of such property over the amount paid for such property shall be included as compensation in the gross income of such employee for the taxable year in which the property becomes substantially vested. 1.83-3(a)Transfer: A transfer of property occurs when a person acquires a beneficial ownership interest in such property 1.83-3(a)(2)Option: The grant of an option to purchase certain property does not constitute a transfer of such property. 1.83-3(b)Substantially Vested and Non-Vested Property: Property is substantially non-vested when it is subject to a substantial risk of forfeiture, and is nontransferable . Property is substantially vested for such purposes when it is either transferable or not subject to a substantial risk of forfeiture. HYPO: On 11/1/78, X corp sells to E, an employee, 100 shares of X corp stock at $10/share, when fair market value is $100/share. Terms: Each share is subject to substantial risk of forfeiture (not lapsing until 11/1/88) and is nontransferable. Since Es stock is substantially non-vested, E does not include any such amount in his gross income as compensation in 1978 On 11/1/88 fair market value is $250/share, and the stock becomes substantially vested. E must include $24K (100 shares of stock x $250 fair market value per share less $10 paid by E for each share). (E paid $1K for stock, fair market value is $25K) 83(b)Election to include in gross income in year of transfer 83(c)(1)Substantial Risk of Forfeiture: The rights of a person in property are subject to a substantial risk of forfeiture is such persons rights to full enjoyment of such property are conditioned upon the future performance of substantial services by any individual. 83(c)(2)Transferability of Property: The rights of a person in property are transferable only if the rights in such property of any transferee are not subject to a substantial risk of forfeiture.

10

1.83-4(b)Basis: If property to which 83 apply is acquired by any person while such property is still substantially non-vested, such persons basis for the property shall reflect any amount paid for such property and any amount includible in the gross income of the person who performed the services. 83(h) allows the employer to deduct the excess of the value of the stock over the exercise price of an Non-Qualified Stock Option. The excess is called the spread. 1.83-6(a)Deduction by Employer: The amount of the deduction is equal to the amount included as compensation in the gross income of the service provider (employee). The deduction is allowed only for the taxable year of that person in which or with which ends taxable year of the service provider in which the amount is included in compensation. (a) Options: Agreement to buy property at the election of the person to whom the option is given. Ordinarily an option must be exercised before a specified date a specified price. Per 83(e)(3), 83 does not apply to the transfer of an option w/out a readily ascertainable fair market value. (b) Non-Qualified Stock Options: Non-Statutory arrangements under 83 which allow the employee to gain immediate ownership of the employees stock at a bargain price and yet give him the possibility of avoiding immediate tax on the bargain purchase (c) Incentive Stock Options: (See 421) Unlike Lobue (non-qualified), an ISO must be given out per a plan adopted by the BOD and approved by S/H. This plan may only go on for 10yrs., but may be extended by the S/H. Must involve an employee receiving an option to purchase the stock of the employer at the fair market value of the employers stock at the date of the grant of the option. (Compare the hypo on page 58) Thus the ISO is only attractive if the price of the employers stock rises after the grant of the option. (i) Advantage of 421: If you follow the rules of the plan, no income when the ISO is granted, and (unlike 83) when you exercise the ISO, you are not taxed. (ii) Disadvantage: Company gets no deduction. HYPO: E, employee receives an ISO allowing the purchase of employer stock for $50/share and exercises the option when the stock has a value of $80/share. The employee has no income, even if the employees rights to the stock are vested. However, unlike NQSOs, the employer which sells stock to an employee who exercises an ISO cannot deduct the difference b/t the stocks value and the price paid. HYPO: E exercises a $10 option when the current value of stock is $50. E then abruptly quits, and calls upon the company to give him $50. To rectify this precarious state of affairs; companies usually have a clause which allows them to repurchase stock @ option price; so employee gets money back that he spent, but not a windfall. (d) Election: Taxpayer may elect to include the bargain element of the transaction as compensation income immediately, rather than following the general rule of deferring inclusion in income until the property has become substantially vested. 83(b) Reg.1.832(a). The election must be made not later than 30 days after the transfer The amount includible would be fair market value of the property, less the price paid for the property.

11

Advantage: If this election is made, the substantial vesting rules will not applyno further amount will be included when the property becomes substantially vested. Any subsequent appreciation in the value of the property is not taxable as compensation to the person who performed the services (Capital Gain) Basis will be amount paid, increased by the amount included in income on its receipt, i.e., amount taxed. Disadvantage: If the property subject to the election is subsequently forfeited back to the employer, no deduction of the amount previously included in income is allowed. HYPO: On 7/1/00, E receives (as compensation) stock of employer, which must be returned to X if she ceases to work for X before 6/30/05. Restriction is noted on the stock certificate and is binding on any transferee. E may transfer the stock (subject to restrictions) by gift or sale. Stock worth7/1/00: $50K 6/30/05: $80K (i) E continues to work for X through 6/30/05. E sells the stock a year later for $100K. E must pay $80K of income tax when it becomes vested on 6/30/05 X may make a deduction when property becomes vested $80K E must then pay $20K on the gain, when he sells. (Basis is 80k) (ii) E makes a timely election under 83(b). E would make an election of $50K After the sale, a gain on subsequent sale of the property at 100k will not be treated as compensatory but will be 50k of capital gain (which is taxed at more favorable rates; usually 20%) (In business transactions, your goal should be turn ordinary income into capital gain. (iii) E makes a timely election under 83(b) but resigns from X in year two and forfeits the stock. No deductions. You are taxed w/out any gain, b/c you made the election. Co. would have to reverse compensation deduction it took earlier. (e) INCOME AND BASIS (when property is received for services) 1012: Basis of property is its cost; you only pay tax on gain over the basis, which is realized HYPO: If A does some work for B and receives a $30K car as compensation, 83(a) requires A to include $30K in gross income. (This is the equivalent of A being paid $30K in cash and using the cash to buy the car). HYPO: A sells the car to C for $30K. Per 1001(b), As amount realized is $30k (the amount of cash received) Whether A realizes a gain depends upon As basis for the car. See 1012property basis is its cost. As gain on selling the car to C must be zero, thus As basis for the car must have been $30K, even though he did not pay cash for it. As cost basis has to be the 30K of income A recognized on receiving the car, not the amount of money (zero) paid for the car. (A is treated as having received $30K of cash as compensation and using it to purchase the car, thus obtaining a $30K basis). As basis for property received as compensation does not arise b/c he invested $30K of his labor in earning the property. (Cost basis of 1012 cant include the value of the taxpayers labor, which economists refer to as opportunity cost) *See problems on page 61

12

(f)

Accounting Methods: Consider a taxpayer who in 2003 becomes entitled to receive a 10K payment for services and who receives the payment in 2004. Whether the payment is includable in the taxpayers 03 or 04 income depends upon the his accounting method. 1. Accrual-Method: Taxpayer reports income upon acquiring a right to the payment *Becomes entitled to a deduction upon incurring liability; not when actually paid.

2. Cash-Method: Taxpayer reports income when cash (property or check) is received. *Most employees use the cash-method to report compensation for services *Normally take-out deductions when they pay out money HYPO: Someone who becomes liable in 2003 to pay a $10K business expense and who actually pays the expense in 2004 will deduct the expense in 2003 if the accrual-method is used and in 2004 if the cashmethod applies. HYPO: T, a cash method, is an attorney whose client owed her $10k for past legal services. The client was in financial difficulty, so T agreed to accept $4k in complete satisfaction of her claim. Can T deduct a 6k loss? Result: NO. As a cash-method taxpayer, T has yet to include anything in income. You may only take a deduction for what is included in income. If T was an accrual-method taxpayer the result if differentT would have already included it w/in income, thus he may be allowed to the deduct the loss. 4. Deferred Compensation: Introduction to the Time Value of Money *Taxpayers can often take advantage of the time-value of money by arranging their affairs so as to postpone tax liability. 1. Goal: The objective of tax planning based on deferral is usually to delay taxes w/out delaying economic gains. (i) Tax-Favored Savings: Because an income tax applies to both a taxpayers earnings from working and to his investment returns, it is sometimes said that the income tax imposes a double tax on those who save some of the money they earn. Consider the following: HYPO (No Tax) W, saved $2k of his salary, and invests it for 20yrs at a return of 10%, compounded annually would yield 13,455 HYPO (TAX-30% on initial earnings and annual investment return) Same facts, 30% of $2K earnings leaves $1400 to be invested. Annual return of 10% is also taxed at 30%, so the annual after tax return is 7%. An investment of $1400 at 7% for 20yrs will grow to $5418. Almost 60 of what taxpayer could have accumulated w/ no tax is. gone. To encourage savings, Congress has enacted a wide variety of programs, basically with two forms of benefit: (a) the earnings of the plan are not taxed at all, or (b) the money contributed to the plan is not taxed when it is earned and the return on the plans investments is not taxed as it accrues, but the original investment and all accumulated savings are taxed when they are w/drawn from the plan. Roth IRA: The amount of contribution is limited to the taxpayers earned income. 3K contribution limit is phased out for taxpayers whose modified AGI exceed the statutory threshold. *Better choice for younger workers *$3000 after-tax limit *Contributions are not deductible, but if the taxpayer takes only qualified distributions from the plan, the earnings of the plan (which grow tax free) are never taxed. *Distributions after 5-year period from first contribution are qualified if they are made after the taxpayer reaches age 59.

13

HYPO: Same facts as above, 30% taxpayer, if he invests by contributing $1400 ($2000 less $600 in tax) to a Roth IRA, the investment will grow to (at 10%) $9418 in 20yrs. At that time, if the taxpayer is over 59 or is otherwise eligible for a qualified distribution, the entire amount can be withdrawn without tax. Without the Roth IRAs tax exemption, the taxpayer would have accumulated only $5418 (after tax) by the end of 20yr. period. Traditional IRA: Two exceptions from the similar eligibility rules and withdrawals to the Roth IRA a) Withdrawals from the traditional IRA can be made w/out losing tax benefits for a wider variety of purposes (medical expenses, educational costs, etc) b) Amount of modified AGI the taxpayer can have before losing eligibility for taxfavored contribution is considerably lower for traditional IRAs *$3000 pre-tax limit *You deduct amount of contributions; when you take outyou are taxedmust take out when you are 70 *Because contributions to and earning of a traditional IRA are eventually subject to income tax, while earnings of a Roth IRA are never taxed, traditional IRA may seem at first glance to be inferior. It is an illusion, though HYPO: 30% taxpayer who earned $2K and plans to invest whatever is left after tax at 10% for 20yrs. In traditional IRA, taxpayers initial investment will be $2K (as the investment is deductible, nothing need to be kept back for tax). 20yrs at 10% = $13,455. If entire amount is w/drawn, the amount will be subject to 30% tax as income, leaving $9418 (the same amount as the Roth) Traditional IRAs deferral of taxes on both the initial contributions and the annual earnings on contributions yields the same benefit as the Roths forgiveness of taxes on the earnings, with no benefit on the initial contributions. ii) Other Tax favored Savings Plans: The real action ($) involves plans having much higher contribution limits. (1) Qualified (deferred compensation) Plans: Meet a variety of explicit rules relating to employee participation and benefits. Governed by 401-404 (See 401Kw/ higher contributions limits) *The earnings are not subject to tax when they are contributed or when they accumulate, but only when distributed/taken out. (build up is tax free) 401(K): Employee can contribute up to 15% of salary; not taxed when contributed or while it builds, only when taken out.

a) Tax Credit for IRA contributions and other retirement plans: 25(b) gives many low-income tax
payers a non-refundable tax credit for contributions. For married couples, filing joint, the credit can be as much as half of the first 2K a year contribution if the couples modified AGI is 30K or less. No credit is allowed couples w/ modified AGI over 50K AGI limits for most unmarried taxpayers are half those for couples. b) Tax Free Roll-overs: When employee is fired, quits or retires, the employee may want to withdraw the funds from the employee plan (i.e., 401k), so that he can direct future investments. Ordinarily, distribution would trigger an income tax on the entire amount, and if under 59, a 10% penalty tax on premature distributions may apply.

14

Rollover would allow funds to be transferred to a traditional IRA, or other qualified plan, within 60 days and avoid tax on the distribution. Rule calls for the funds to be rolled over directly to the receiving plan, rather than being distributed to the taxpayer, then transferring. (A 20% withholding tax may be assessed-3405(c).) If taxed per 3405, employee must come up with some other source of income to avoid being taxed on part of the distribution. HYPO: $1M, is employees distribution, if it is paid to the employee, who then transfers to the IRA, the withholding tax will take $200K, so the employee will receive only $800K. Because a completely taxfree rollover must consist of the entire amount of the distribution, the employee will have to come up with $200K, to reinvest the full $1M c) Qualified Domestic Relations Order: Divorcing couples cannot simply assign rights to benefits under pension or profit sharing plans by agreement. The state court must enter a decree that is a QDRO in order to transfer interest (2) Non-Qualified, Contractual (deferred compensation) Plans: Setting aside of money for deferred taxation, which does not have to meet broad requirements w/ respect to employee participation, benefits and funding. Employer generally must be willing to undergo a tax disadvantage to confer unusual benefits on the object of its bounty. Employer is subsidizing the employees tax deferralemployer must still pay taxes on the money. *The number of nonqualified deferral plans has exploded b/c of the increased restrictions enacted w/ respect to qualified planssee ERISA, limiting the amount how much executives could save in qualified plans Constructive Receipt: T.Reg. 1.451-2(a) provides that income, although not actually reduced to a taxpayers possession, is constructively received by him in the year in which it is credited to his account, set apart for him, or otherwise made available so that he may draw upon it at any time. However, income is not constructively received if the taxpayers control of its receipt is subject to substantial limitations or restrictions HYPO: On 12/30/01, E tells employer to not give him the 10k at this point. Wait till next year b/c I had a real good year this year, and I might need it next year. Per 1.451-2, he constructively received it on 12-30.

i)

ii)

Solution: If at the beginning of the year you tell employer not pay you 10k of your 100k salary, instead paying you the 10k in 10yrs--it is not constructively received (even if employer puts into a bank account) if: (1) The money is not subject to you taking it; you have no right to demand it. (2) The money is subject to the creditors of the employer.

5. ILLEGAL GAINS T.Reg. 1.614(a) gross income includes illegal gains, and a deduction is allowed if and when the wrongdoer reimburses the victim for the loss per 165(a) and 165(c)(2) as a transaction entered into for profit. James v. US F: Union official embezzled in excess of $738K in 3yrs., and did not report these amounts in gross income. Convicted of tax evasion. I: Are embezzled funds to be included in gross income of the embezzler in the year in which the funds are misappropriated

15

Wilcox decision was on the books and said that embezzled funds were not income b/c a taxable gain is conditioned upon a) the presence of a claim of right to the alleged gain, and b) the absence of a definite, unconditional obligation to repay or return. Rutkin decision was also on the books which said that extorted funds were income. H: 1) Overruled Wilcox, illegal income is taxable despite the recipients legal obligation to make restitution. Court uses language similar to Glenshaw Glass, i.e., it is an accession to wealth. 2)The prosecution of the taxpayer for tax fraud, however, is overruled. Fraud requires willfulness and b/c of Wilcox being on the books at the time the alleged crime was committed, this could not have been considered willful. (James paints with a broad brushall illegal income is taxable) D(1): Didnt he read Rutkin also, which found extorted funds to be income D(2): Remand to see if really did rely upon Wilcox HYPO: T is trustee of a family trust which is not subject to court supervision. T, in breach of the trust, sells to himself for 4k, a trust asset worth 10k. What result? 6k gain to T.

a. Victim v. Government: Who has the superior claim? The governments interest in taxing this as
income, or the victims interest in reimbursement? The victims right to reimbursement is sometimes subordinated to the governments claim for taxes. This is offensive, considering that the taxpayer may then become judgment proof. Loans: Loans are generally not income. If they were income, repayments would be deductions and be very complicated. In James, there was no consensual obligation to re-pay, therefore, this could not be considered a loan. If you are caught embezzling, you could not simply say, Ill repay you, this is just a loan. Furthermore, if the T purports to borrow funds for a legitimate business purpose, but is actually swindling the lenders, if the intent to cheat is dominant, the funds are taxable fruits of larceny or embezzlement. (If the investment faade is given credence, the receipts are nontaxable borrowed funds). Tax Crimes: 7201-7216Use of these provisions to enforce tax laws resembles enforcement of traffic laws in the sense that the government makes no serious attempt to detect and convict every tax criminal. Only the most flagrant offenders are prosecuted The tax law and non-tax criminals: Rutkin and James gives the federal government power to punish local crimes such as embezzlement and extortion (by including illegal gains as income subject to 1040 reporting). Self-incrimination and tax returns: Willful failure to file a return, or to pay a tax, is a crime, and an accurate return must disclose both the amount and the source of the taxpayers income. This suggests that a non-tax criminal who wants to avoid becoming a tax criminal too should file a tax return reporting income from illegal activities, describing those activities briefly. Which may lead to an investigation of the taxpayer. Some law says that you can just list as income 100K, not revealing source, then invoke 5th. But the 5th cannot be used to justify failure to file on the theory that filing by a criminal would be incriminatory. You must report

b.

c.

d. e.

HYPO: T is trustee of a family trust which is not subject to court supervision. T, in breach of trust, sells to himself for $4K a trust asset worth $10K. What result to T. 6k gain is taxable. Basis is now $10K This is not a bargain purchase or a windfall. You treat people differently by the way they act.

16

C. RECOVERY OF CAPITAL Measuring income the sense of gain or profit requires making an allowance for the costs of earning income. The following material will highlight that not all receipts are gains. a. Sales: In the case of a sale, the taxpayers recovery of capital takes place at the stage of calculating gross income. This is the simplest form of capital recovery. HYPO: E buys corporate stock for $3k and sells it for $3700. Es gross income is $700. His cost basis is $3k, which is not taxable, b/c it is a recovery of capital.

b. Dividends, Interest and Rent: These are taxable, w/out regard to any cost basis. HYPO: T spends $100K to buy a tract of farm land, which he then leases for $8K per year. The $8K rent is includable in the taxpayers gross income each year. The taxpayers $100K investmenthis basis does not enter into the calculation of either the gross income from the property or taxable income, until he sells the land. HYPO: T buys corporate stock for 100K. He cannot use any of the stocks 100K basis to offset the receipt of cash dividends; as the previous hypo indicates, the use of basis must await he disposition of the stock c. Depreciation: Largest amount of deductions, next to salary. 167 permits the taxpayer to
recover (tax-free) the cost of property that is exhausted in the process of generating business income. Property may not be depreciated unless the Ts investment will be consumed, in part or whole, by the effects of exhaustion, wear and tear, or obsolescence. 1) Concept: Assets used in business become less valuable as time goes by. Thus a taxpayer owning the asset deducts annually an arbitrary percentage of the cost of the asset. (If the asset is an intangible asset, such as a copyright, the deduction is known as amortization) *Depreciation is most often utilized as the tool for which government re-formulates economic policy. 2) Straight-Line Depreciation: Depreciate by taking deduction each year for % decreased. 3) Wasting Investment Requirement: Depreciation is denied for assets that are not adversely affected by the passage of time or by use in the Ts business, such as works of art, antiques, and raw land. However, courts have held that depreciation is allowed for property having value as a work of art if the property sustains wear and tear in its use in the Ts trade or business. (See Simon v. Commn, allowing violins to be depreciated, b/c their use subjected them to wear and tear. The court rejected the argument that proof of a determinable useful life is an essential precondition to the depreciation deduction. They said that 168s purpose was to relieve Ts of the burden of establishing useful life. This was allowed e/t the violins apparently appreciated in value as the Ts used them.) IRC 167(a)General Rule: There shall be allowed as a depreciation deduction a reasonable allowance for the exhaustion, wear & tear (including reasonable allowance for obsolescence (1) of property used in the trade or business, or (2) of property held for the production of income. Thus, a machine or computer would be the type of property subject to depreciation, but not a raw piece of land. Your own personal car, computer, or house will not qualify unless used in a trade or business and for production of income. The basic legislative policy permits deduction for the cost of making a living, but not the cost of living.

17

(c)Basis for Depreciation (1) In generalthe basis on which exhaustion, wear and tear are to be allowed in respect of any property shall be the adjusted basis provided in 1011, 1016(a)(2) for the purpose of determining the gain on the sale or other disposition of such property. HYPO: A taxpayer buys a truck for 10k, takes a straight line depreciation of $4k, and sells the truck for $7k. What is Ts gain After the depreciation deduction, his adjusted basis was $6k, Gain is $1k IRC 168: If property qualifies under 167, use this section, which is primarily accountants work. This section solves problems by limiting salvage value, which essentially asked the questions of how long will the asset last and how much is an asset was worth at the end of its life? HYPO: If a taxpayer pays 50K for an asset which will be used to produce income for 5yrs., and which will then be worthless, must at some point deduct 50K from receipts if only gain is to be taxed. The taxpayer would deduct 1/5 ($10K) of the assets cost each year for five yrs. Unlike the sale of property, the money the taxpayer paid (for the asset) does not affect the determination of gross income Hampton Pontiac v. United States: F: P entered into an agreement w/ King Pontiacs controlling S/H (Matthews) that King Pontiac would surrender its franchise, and Matthews would receive money. The money paid to Matthews was deducted by P as a salary expense on its federal tax return. The deduction was disallowed, P paid the disallowed deduction and now requests a refund. H: R: 1) The payments were capital investments, not deductible as ordinary and necessary business expenses. The contract w/ Matthews was an integral part of Ps expense in acquiring its Pontiac Franchise. Payments were includible in Ps invested capital. 2) The capital investments are not amortizable (under 167 of the IRC, which allows intangible assets to be depreciated if the useful life of the asset is of limited duration, capable from experience of being estimated w/ reasonable accuracy) over the initial fixed term of the franchise agreement (i.e., five years). The P has failed to bear the burden of establishing the amortizable character of his payment on the basis of the fixed life of the franchise. Giving due weight to the actual record of non-renewals of GM franchises, it cannot be said that the enjoyment of this franchise may with reason be expected to end in five years. Quite contrary, the franchise was reasonably certain of renewal. Safeguards were in place review by former SC Justice. 3) But such payments may be amortized over the life expectancy of Samuel W. Jones, as computed by the mortality tables issued under the Treasury Reg. It is clear from the records that the death of the participating owner will normally occasion the termination of the franchise 4) Notes on Depreciation: a) Professional Sports Owner: The owner of a professional sports franchise is allowed to depreciate the K of his players. Does the owner get a double benefit, considering that he gets a depreciation deduction and a deduction for the players salary (a business expense under 162) Should the players get a deduction? The athlete may not, considering that he has no basis. b) HYPO: Taxpayer buys a truck for $10K, takes a straight line depreciation of $4K, and sells the truck for $7K. Ts basis is reduced to $6K, sold for $7K, thus he has a $1K gain. c) Depreciation accounts for the largest amount of deduction, next to salary.

18

d) Property Having unlimited useful life: Tangible assets that do not wear out, cannot be depreciated. e) However, musicians have been able to depreciate rare musical instruments, even though such instruments generally increase in value and last for hundreds of years, b/c they do suffer wear and tear w/ use.

d. Annuities: Contract providing for regular payments (after you pay a single or series of
premiums) from an insurance Co., to the annuitant, with an interest element, which begin at retirement or other fixed date and continue for a term of years or for the life of one or more individuals. (Essentially assuring the recipient of a lifetime source of funds) Insurance Co. could go bankrupt. Insurance Co. is willing to bet that they make more on your premiums than they will have to pay you back in interest. What you are paid back depends upon what interest rate the Co. is willing to assume. What you are getting back is not just interest, but part of your principle (invested capital) you put in plus the interest. Thus the question becomeswhat are you taxed on? IRC 72(b): Gross income does not include that part of any amount received as an annuity which bears the same ratio to such amount as the investment in the contract, i.e., the portion of return which represents your premiums is after tax dollars, which is not taxed again (Only taxed on the interest you receive) Thus we need to determine your exclusion ratio, which you will not be taxed on.

i)

Exclusion Ratio: This ratio is determined by dividing the investment in the K as found under 1.72-6 by the expected return under such K as found under 1.72-5. Investment in K Expected Return

Exclusion Ration=

The exclusion ratio for the particular K is then applied to the total amount received as an annuity during the taxable year by each recipient. Any excess of the total amount received (during the taxable year) over the amount determined to be the exclusion ratio, shall be included in the gross income of the recipient for the taxable year of receipt. HYPO: Taxpayer A purchased an annuity K providing payments of $100 per month for consideration of $12,650. Assume the expected return is $16,000. The exclusion ratio to be used by A is $12,650/$16,000; or 79.1 percent. If 12 such monthly payments are received by A during his taxable year, the total he may exclude from gross income in such year is $949.20 ($1200 x 79.1) The balance ($1,200-$949.20) $250.80 is gross income.

ii)

Expected Return: (If a K to which 72 applies provides that one annuitant is to receive a fixed monthly income for life) the expected return is determined by multiplying the total of the annuity payments to be received annually by the multiple shown in Table I or V of 1.72-9 under the age (as of the annuity starting date) and, if applicable, sex of the measuring life (usually the annuitants)

HYPO: Male, 66, purchases a K prior to July 1, 1986, providing for annuity of $100/month for his life. Monthly payment of 100 x 12 months equals annual payment of.$1200 Multiple shown in Table I male, age 6614.4

19

Expected return (1,200 x 14.4).$17,280 HYPO (a): A, who is 65 on 1/1/01 pays an insurance company $144K for an annuity of $1000 a month for As lifetime, payments to begin on 1/31/01. How much is income? Investment: $144,000 Expected Return: 240,000 (12,000 x 20.0) Exclusion Ratio: 60% (144,000/240,000) 12,000 x 60%=$7,200 12,000-7200=$4,800 Include as income Mortality Loss: The annuitant dies before life expectancy HYPO (b): In January 2003, A died. A received $24K in annuity payments in year 1 & 2, and nothing in year 3. What is As deduction for year 3? $129,600. Per 72(b)(3), the amount of unrecovered investment is deductible (144,000) reduced by the aggregate amount received under the K, which was not includible in gross income. ($14,400) (he received $24K in two years, and each year $7200, was not includible in income this amount is counted towards his recovering his investment). Estate will actually get to make this deduction. Mortality Gain: The annuitant outlives his life expectancy HYPO: If the reverse happens, and the guy lives much past 20yrs., then everything after his life expectancy is gravy. All must be included as income. iii) Deferred Annuities: This will yield a similar tax benefit as the non-deductible contribution to a traditional IRA. The main difference is that there is no statutory limit on the amount that can be invested in an annuity K. One drawback is that most taxpayers who w/draw lump sums before reaching age 59 become subject to an extra early w/drawl tax of 10% 4. CANCELLATION OF INDEBTEDNESS General Rule on Borrowing: Before you get out of debt, you have to get in debt by borrowing. As a general rule, borrowing does not give rise to income to the borrower and repaying the debt does not gives rise to a deduction. As for the lender, he does not have a deduction when making the loan, and does not have income on repayment. Borrowing creates an immediate offsetting obligation to repay and therefore the borrower is not wealthier than he was before borrowing, The better explanation of why there is no income is that the amount realized is the amount of the loan, and his basis is also the amount of the loan. Thus there is no gain realized, since amount realized equals basis. This indicates that an increase in wealth is not necessarily the touchstone of taxation.

a) Discharge of Indebtedness: When a person debt is cancelled or reduced, his net worth is
increased, and the amount reduced or cancelled is treated as income, unless if falls w/in several possible exceptions. See IRC 61(a)(12): Income from discharge of indebtedness. United States v. Kirby Lumber Co. F: Kirby issues $12M of its own bonds for which it received their par value. Later in the same year, it purchased in the open market about $1M of the same bonds for $862K, less than par, the difference in price being 137K less than what the company had sold them for. (This was possible b/c interest rates had risen in the economy, driving bond prices down b/c richer yields are available throughout the markets).

20

H: The excess of the issuing price or face value, over the purchase price is gain or income for the taxable year. Their balance sheet became healthier b/c they do need to pay back the 137K R: Unlike Kerbaugh-Empire, there was no shrinkage of assets and the taxpayer made a clear gain; accession to wealth. As a result of its dealings, it made available 137K in assets previously offset by the obligation of bonds now extinct. The will not have to pay back this 137K HYPO: If the bank borrows me 10K, and I blow all of it at the casino, and the bank forgives the debt, Kerbaugh-Empire implies that this is not income b/c the transaction (at the casino) was a total loss. This is why Kerbaugh-Empire is not a reliable case

b) Insolvency Exception: The Courts have created a doctrine under which cancellation-ofindebtedness income is taxed only to the extent that the taxpayer is solvent after the cancellation. IRC 108(a)(1): Gross income does not include any amountby reason of discharge of indebtedness of the taxpayer if (A) Discharge occurs in a bankruptcy case (B) Discharge occurs when the taxpayer is insolvent (Note that he bankruptcy exception goes further than the insolvency exception by excluding cancellations of debt in bankruptcy in full, even if the debtor becomes solvent as a result of the proceedings) Per 108(a)(3): The amount excluded shall not exceed the amount by which you are insolvent. Thus, if you are 10k insolvent, and 20k in debtyou can only exclude 10k

i)

Who is Insolvent? 108(d)(3): The term insolvent means the excess of liabilities over the fair market value of assets, without qualifying Carlson v. Commn: Held that assets as defined in 108(d)(3) includes all of the taxpayers assets, not just non-exempt assets HYPO: T owes $100K in debts and has two assets, each worth $75K. One of the assets is exempt from the claims of creditors; the other is not. In determining if T is insolvent, 108(d)(3) seems to indicate that T has assets of 150K (by including the exempt asset)

c) Reduction of basis following exclusion of cancellation of indebtedness 108(b) requires that the taxpayer apply the exclusions under 108 (a) by reducing the tax attributes of the taxpayer in the order so provided in 108(b) One tax attribute is the basis of the property held by the taxpayer However, the basis of exempt property (under the Bankruptcy Act) will not be reduced 1017 contains directions for calculating basis reduction (made at the beginning of the taxable year following the taxable year in which the discharge occurs) Another tax attribute is net-operating carryover: a loss from an earlier year which, b/c of its deduction in that year would provide no benefit, can be carried over to future years and deducted against those years incomes HYPO: Before a debt cancellation, Ts assets are worth $1000, and her liabilities total $1600. T is insolvent. One of her debts, in the amount of $900 is canceled upon payment of $100. (Ordinarily this would result in debt cancellation income of $800). However, T is insolvent (b/c the discharge occurred when T was insolvent) to the extent of $600, and so $600 of the income is not taxed. Consequently, Ts income is only $200 T must reduce tax benefits by the remaining $600 per 108(b). T must go down the list of 108(b) until she finds tax attributes sufficient to absorb the entire amount of untaxed income.

21

d) Liabilities giving rise to Deductions:


Per 108(e)(2) No income shall be realized from the discharge of indebtedness to the extent that payment of the liability would have given rise to a deduction. HYPO: F, cash method lawyer, owes G $500 for services G has performed for F. F would be entitled to a $500 deduction upon paying for these services. G cancels $300 of the debt. $300 is not income to F b/c the payment of the liability would have given rise to a deduction.

e) RAIL-JOINT PRINCIPLE: Several courts have held that cancellation of a debt gives rise to
income only if incurring the debt gave the taxpayer money, property, or services. Commissioner v. Rail Joint: Company distributed its own bonds to its S/H as a dividend, then bought back some of the bonds for less than face value. Held, Corp., having received nothing when it issued the bonds, (i.e., when it incurred debt they did not get $, property, services) had no income when it bought them back for less than face value. The Rail Joint court distinguish Kirby Lumber by saying that Kirbys assets were increased by the cash received for the bonds

f) Reduction of Purchase Price:


See 108(e)(5) If the debt of a purchaser of property (to the seller of such property) which arose out of the purchase of such property is reduced, such reduction does not occur in a bankruptcy case or when the purchaser is insolvent. Statutory reduction of purchase price rule applies only in the case of a debt owed to the seller of property. Only applies b/t a creditor and purchaser -Hirsch v. Commn: T bought property for 29K (paying 10K down, assuming 19K mortgage debt) Several years later, the value of the property fell to 8K. The mortgagee agreed to take 8K in full satisfaction. Held, the $7K that was cancelled was a reduction in the purchase price and not income. HYPO: Buck Gray car purchase, you paid 10k, find out some things are wrong, Buck says Ill give it to you for 7k. This is only an adjustment of the purchase price.

g) Qualified Real Property Business Indebtedness: An exclusion is created under 108(a)(1)(D), for
taxpayers, other than C corps, and applies only to the extent that the amount of the debt before cancellation exceeds the value of the property which secures the debt. You must have depreciable property. To wit: HYPO: T incurs a debt of 200K to buy a building used in a trade, and the value of the bldg. falls to $180K. A cancellation of 30K, will produce only 10K of income. (108(a)(1)(D) only applies to the amount the debt exceeds the value, i.e., 20K.) The basis of the property must be reduced by the amount of debt-cancellation income excluded, i.e., 20K. (see 108(c)(1)(A), which provides that the basis of depreciable real property must be reduced by the amount excluded from gross income. Revenue Ruling 84-176 F: Corp. (buyer) entered into K w/ seller, under which buyer agreed to purchase various quantities of goods. Seller shipped some goods, but refused to ship the rest. Buyer refuses to pay ($1K) for the goods already received and files a counterclaim. Settlementbuyer agreed to pay $500, remaining 500 was forgiven, in return of buyer dropping counterclaim. Seller excluded $500 from income per 108 and reduced the basis of its assets by that amount as reqd by 107. H: This amount that the seller waived, is to be treated as payment for lost profits rather than a discharge of indebtedness.

22

R: Debt discharge that is only a medium for some other form of payment, such as a gift or salary, is treated as that form of payment rather than under the debt discharge rules (Not every indebtedness that is cancelled results in gross income being realized by the debtor by reason of discharge of indebtedness. If a cancellation of indebtedness is simply the medium for payment of some other form of income)

h) Loan-Forgiveness Programs: Many schools have adopted programs designed to encourage their
graduates to take low-paying public interest jobs, by forgiving the loans from the school as the graduate works. Under 108(f)(3), the exclusion does not apply to loan-forgiveness on account of services performed for the educational organization itself. **If not w/in a 108 exclusion, it is income. 5. DAMAGES AND SOCIAL WELFARE PAYMENTS a. Damages in General: 104if payment does not arise out of physical injury, utilize in lieu of test. Thus, Age and Sex discrimination, libel, slander, etc are all includable in income. Attorneys Fees: Money you expend to produce taxable income is normally deductible (below the line) See 212. Sager Glove Corp. v. Commn F: Sager sued Bausch & Lomb Optical for violation of Antitrust violations (refusing to supply it w/ lenses for safety goggles), which forced Sager to cancel a K w/ General Motors, lose customers, etc Case settled for $478K. 132K was paid as reimbursement for atty. fees, to which constituted ordinary incomewhich is deductible. I: What is the remaining $318K? Sager contends that 1) Punitives which are not taxable, and 2) it is nontaxable return of capital (see footnote #2, pg. 142, if no basis, a return of capital would be capital gain. H: Taxable as ordinary income, as it looks like compensation for lost profits R: If the recovery represents damages for lost profits, it is taxable as ordinary income. However, if it represents a replacement of capital destroyed or injured, the money received, to the extent it does not exceed the basis, is return of capital and not taxable.

i)

Taxation of Damages: In Raytheon Production Corp. v. Commn the 1st Cir. put the question of whether damages are taxable as: In lieu of what were the damages awarded? It is often suggested that the wording of the taxpayers complaint in the suit for damages is controlling Compensation for an injury to Goodwill, is not taxable, while loss of profits are taxable. ii) Business and Personal Injuries: Damages for the destruction of property (personal or business) which exceed the basis of the property are income. HYPO: Vacation home has basis of 20K, and is destroyed. Taxpayer receives 30K in damages from tortfeasor or insurance payments. T has 10K of income.

1)

INCOME UNDER 104

23

Pre 1996 104: Under 104(a)(2): all of the compensatory damageslost wages and pain and sufferingwere excludable. a) Present day 104: In Generalexcept in the case of amount attributable to (and not in excess of) deductions allowed under 213 (relating to medical, etc., expenses) for any prior taxable year, gross income does not include (1) Workers Compensation (2) Damages (other than punitives) received (whether by suit or agreement and whether as lump sums or as periodic payments) on account of personal physical injuries or physical sickness. For purposes of para. (2), emotional distress shall not be treated as a physical injury or physical sickness. If not a physical injury, go back to the in lieu of test. Injury must come from outside agency, not self-inflicted -i.e., a doctor jumping of bridge b/c of libel -i.e., a headache, anxietytough rocks Mental/Psychological Injury is not included w/in 104 unless it is the result of a physical injury HYPO: Son of M, almost gets hit by a car, and now has headaches, etcNo physical injury to the son. HYPO: If car hit Son, causing physical injury (i.e., contusions, bruises) and M recovers, this is excludable, b/c event which gave rise to award was physical injury. HYPO: M is accused of sexual harassment by S. S sues M and wins. This is not excludable b/c there was not physical injury. Two Fundamental kinds of damages: Damage to the person and to the business. b) Damages from Personal Injuries Compensatory damages received on account of personal physical injuries or physical sickness are generally excluded. See 104(2)(a) Punitive damages are always included. Emotional distress is not a physical injury or physical sickness, though recoveries for medical expenses in emotional distress cases remain excludable. See 104(c) c) Damages from Injury to Business d) Annuity Payments (Structured Settlements): An accident victim who receives 100K from an insurance Co. and uses the 100K to buy an annuity, includes the interest portion of the annuity in income under 72. However, if the settlement takes the economically identical form of an annuity (periodic) payment to the victim at an increased %, the victim can exclude all of the annuity payment. Must be labeled as an interest component of an annuity received as personal injury damages, not as interest in substance. HYPO: 300k settlement (right now) or 360k spread over a period of time? If you took the 300k up front and bought an annuity, part of it is taxable. However, if you took the 360k, none of it is includable in income, e/t there is an interest element. 6. IMPUTED INCOME: Benefits created when a taxpayer performs services for himself (or his family). If you paint your own house, no income is gained by not having to pay a painter. Williams v. Commn W was paid 10% commission on each lot he sold. He purchased 10 of the lots, and got a check for 10% of the purchase price on each lot he bought.

24

H: Income. What the idiots should have done is reduced the purchase prices by the amount of his commission. (132) Chapter 3: Exclusions, Deductions, and Credits A. Interest on State and Local Obligations: 103: Excludes from income interest on the debt obligations of states and their political subdivisions B. Gifts: Ever since 1913, gifts and inheritances have been excluded from income. This exclusion raises some puzzling theoretical questions. One who receives a gift or inheritance has certainly enjoyed an increase in wealth (See Glenshaw), but is nonetheless exempt. *102: Gross Income does not include the value of property acquired by gift, bequest, devise, or inheritance *Donors get no deductions for gifts and donees have no income: If A earns a salary or collects interest on a bond and then gives some of the salary or interest to B, A has income (per 61) and B does not (via 102). (1) What qualifies as a gift? The most critical consideration is the transferors intention. The donors characterization of his action is not determinative. Mere absence of a legal or moral obligation to make a payment does not establish that it is a gift. See Old Colony A gift in the statutory sense, proceeds from a detached and disinterested generosity. See Lobue Per Duberstein, the reach of the concept of income is pretty broad when gratuitous transfers are made in a business setting. Commn v. Duberstein F: 1) Duberstein, pres. of Duberstein Iron & Metal Co., referred customers to Berman, pres. of Mohawk Metal Co., in the course of doing business. In 1951, Berman gave Duberstein a present of a Cadillac b/c the info Duberstein had given was so helpful. No obligation, just decided to. Mohawk deducted the Caddy as a business expense. 2) Stanton: Employee gave payment to retiring employee H: 1)Tax Court was not clearly erroneous in finding Caddy not to be a gift, b/c no intention of gift, but a remuneration for services provided or inducement of future services. 2) Remanded, b/c Dist. Courts conclusion was a general finding that it was a gift, w/ no concrete indication of the legal standard utilized. R: Transferors intention is of paramount significance; what is the dominant reason that explains his action in making the transaction. The test under Duberstein is the Donors intent, not what the donee may have thought. Taxing statute does not make non-deductibility by the transferor a condition on the gift exclusion Decision as to whether it is a gift must be based on trier-of-facts review of totality of facts in each specific case. Give deference to the trier of fact. HYPO: M gives 10k to his smoking daughter. Income? No. Probably a gift, as this is not really compensation for services. Per Duberstein, look into Ms head. Did he intend gift? HYPO: M, thinking that P is underpaid, gives P (local priest) $50. M is augmenting Ps income. The more the priest starts thinking this payment to be an augmentation of services, and expects itthe more clearly it becomes income. *The family relationship in hypo #1 provides the distinction from hypo #2.

25

United States v. Harris: F: K, deceased, was a wealthy widower, whom gave two broads each more than half a million dollars over the course of several years. Is this income to the two ladies? H: Remand w/ instructions to reverse indictments against the two ladies. R: How do you ascertain the intent of the dead guy? (2) Gifts to Employees: 102(c), enacted in 1986, provides a conclusive presumption of income. In general, there shall not be excluded from income any amount transferred by an employer to an employee; except for de minimis gifts under 132, and some non-cash achievement awards under 74(c). Overrules Stanton: A gift to employees for long and loyal service, constitutes income. But see proposed T.Reg. 1.102-1, which proposes that 102(c) should not be applied to amount transferred b/c related parties (e.g., father & son) if the purpose of the transfer can be substantially attributed to the familial relationship (natural object of bounty)of the parties and not the circumstances of employment. (3) Gifts of Income from Property: Instead of an outright gift of property, the donor may confer the income from property as a gift. In such an event, the income from the property is taxable to the donee, notwithstanding that the transaction is indisputably a gift. See 102(b) (including income from any property) *If the gift takes the form of periodic payments from specified property, it is taxable to the recipient to the extent the payments are out of income from the property and excludable to the extent the payments must be made from principal. See 102(b) (4) Bequests/Inheritances: Like gifts, bequests, devises, inheritances are excluded from income by 102. A spouses statutory share is not income, even if decedent tired to disinherit BASIS: The basis of inherited property is the propertys value at the date of the decedents death. 1014no tax on the amount inherited, but you get a stepped-up basis (5) Basis of Gifts & Bequests (A) Gifts: 1015 applies to transfers by gift. (3) General Rule is Carryover Basisthe donee takes the donors basis for determining gain or loss HYPO: A transfers stock to B as a gift Basis of Donor (A)$10K FMV$15K Donees (B) basis$10K If B then transfers to C, and the FMV is $20K, Cs basis (as long as it is a 102 transfer) would be $10K When this stock is sold, gain will be taxed (low basis, but higher FMV). HYPO: If A sold for its FMV (15K), he would pay tax upon the $5k gain, then give it to B (whom would have a $15k basis although basis isnt usually talked about in terms of cash)

(4) Exception: If FMV is less than basis at time of gift, for purposes of
determining loss, basis shall be FMV. HYPO:

26

Basis of Donor$10K FMV$8K Sale by Donee$9K (No gain or loss) Sale by Donee$11K ($1K gain) Sale by Donee$6K ($2K loss)

(B) Bequests: 1014 applies to property acquired from decedent (1) General Rule is Stepped-up Basisthe person acquiring the
property takes, as their basis, the FMV of the property at the date of death or some other valuation date. (2) This could produce a step-down if FMV was less than decedents basis. HYPO: A has 250K property, and takes 200K in depreciation deductions, thus basis is now 50K. When A dies, the FMV of his property is 450K. The property is willed to B, B basis in the property is 450K. If B takes 200K in deductions, his new basis is 250K. HYPO: Old man Swygert buys Lake Michigan house for 28K in 1930. Today it is worth $1.2M. Old man says he is going to give it to Mike Swygert. Mike says Dad, dont give the property to me now, as a gift, b/c my basis will be $28K Rather transfer it to me at death, and then my basis will be $1.2M C. Prizes: Prizes and Awards are included in income under 74(a). 1. Exceptions: (a) Charitable Transfer: A prize is not taxable if (i) it was received for scientific, educational, or similar achievements, and (ii) the recipient orders that it be transferred to a charity. See 74(b) (recipient would also not be entitled to a charitable deduction for the transfer) (b) Employee Achievement Awards: An award to an employee for length of service or safety achievement is not taxable unless it cost the employee more than $400. See 74(c), 274(j) HYPO: T, a professional football player, received an automobile from a sports magazine after he was named the MVP in the championship game. Held to be a taxable accession to wealth under Glenshaw. This was not a tax-free gift under 102, nor was it an award for noteworthy service under 102. Turner v. Commn F: R was called on phone and asked to name a song that was being played on the radio. He and his wife received tickets for a cruise to Buenos Aires. The couple reported income of $520, representing income of the two tickets. Commn increased the income to $2,220, the retail of such tickets. Tax Court said value is $1400 w/ no apparent rationale as to why. D. Life-Insurance Proceeds: As a broad general rule, proceeds from life insurance contracts are taxable, however, proceeds paid under an insurance contract, by reason of the death of the insured, are excluded from gross income [I.R.C. 101(a)][ 264(a)(1) No deduction shall be allowed for premiums on any life insurance policy, or annuity contract if taxpayer is directly or indirectly a beneficiary under the policy or K] 1) Two major (huge) advantages of life insurance (a) All proceeds are tax-free (b) Investment element builds up tax-free (similar to Roth IRA)

27

(Life insurance companies bring important long term investment strategies to the capital markets) 2) Basic Concepts: a) Premiums depend upon your insurability (age, disease, etc) b) Owner of policy (whom may cancel coverage, change beneficiary, etc) may be the insured or somebody else c) Life insurance company wants you to live forever so that you pay more in premiums than you receive from death benefits. Pure gamble that insured will live longif the insured dies young the Co. loses. (Compare an annuity, where they want you to die right away)

3) Term Insurance: No investment feature, simply provides death benefits if the insured dies w/in
the coverage period usually for a set term, usually 5-10 yrs. a) Renewability: Allows you to purchase for another term (at a higher rate b/c you are older) w/out going through insurability process, i.e., taking a physical exam. b) Convertability: Some term-insurance policies may be converted by the insured to a more-elaborate kind of policy. c) Decreasing:

4) Group-Term Insurance: Provided to a group of employees and provided under a policy carried
directly or indirectly by an employer. These benefits must be non-discriminatory, in that the CEO automatically gets X, while the janitor gets Y. 79: Provides that the first $50K is not taxable. Thereafter, it must be included in gross income. *This Corporate Owned Life Insurance (COLI) is said to cost taxpayers 11.9B a year in lost revenue (This may be key-term) 5) Whole Life Insurance: Premium you pay$1000. $100 goes to the mortality coverage (term), and $900 goes into an investment account. Systematic plan As $900 grows, none of the build-up is taxable. a) Insurance/Investment: 7702 requires that if you have a life insurance policy that produces investment, it must have a significant insurance element. (Not a very restrictive test) b) Cash Value: Life insurance policies combining term and investment components, provide ways in which the insured can w/draw the accumulated investment A common use of the return on the investment component of life insurance is to pay some of the premiums on the policyresulting in a complete tax exemption for the investment return ( cash value) b/c 72(e)(4)(b) provides that dividends on insurance policies are tax free if retained by the insurer as premiums. (Contrast someone who buys term ins., and pays the premiums w/ interest earned on a bank account). As a rule, lump-sum payments to an insured (amount not received as annuity) are tax free until the total amount of the payments to the insured exceeds the total amount of premiums paid by the insured. (By contrast, early w/drawl from annuity K are taxed under 72(e)(2)). The proceeds of loans against insurance policies are tax free b/c loan proceeds are not income.

28

For families, these cash value policies are often a poor deal, b/c the commissions are usually high. But employers can negotiate low commissions and it makes sense for them to round up cash they might have lying around (on which theyre earning taxable returns, and blow it into a life-insurance policy where the returns arent taxed.

6) Transfers for value: In an apparent attempt to discourage trading insurance policies, 101(a)(2)
denies/limits the exclusion in the case of transfers of life insurance contracts for valuable consideration. In the case of a transfer of a life insurance policythe proceeds attributable to such policy that are excludable from the transferees gross income is limited to (i) the actual value of the consideration of such transfer, and (ii) the premiums and other amounts subsequently paid by the transferee However, this limitation on the amount excludable from the transferees gross income does not apply in two circumstances. See 101(a)(2) (A) Where the basis of the policy or interest transferred is determinable (in whole or in part) by reference to the basis of such policy in the hands of the transferor, or (B) Where the policy or interest therein is transferred to the insured, to a partner of the insured, to a partnership in which the insured is a partner, or to a corporation in which the insured is a S/H or officer. HYPO: D sells her policy to S for $8k at a time when Ds basis in the K is $5k (represented by premiums). After S pays 5 yrs worth of premiums, D dies, S receives 100K. *Result to D: $3k gain *Result to S: Must include in income less what he paid for the transfer (8k) and what he paid for in the premiums (5yrs worth).

7) Accelerated death benefits: The insured with a terminal illness or whom is chronically ill
may be able to sell the policy to the company, whom will pay the ill tax-free proceeds before you die. (The insurance co. is betting that the when you die, amount paid to beneficiary is less than what they paid for it) See 101(g) treats any payment to a chronically ill/terminally ill person as an amount paid under the life insurance contract by reason of the death of such insured. Usually lifetime benefits and gains from selling polices are taxed, but 101(g) takes care of it out of an unbridled showing of generosity and humanity.

8) Taxation of Life Insurance: When a policy combines term insurance and an investment
component, one can look at the payment that will go to the survivor as consisting of two parts: (a) an insurance payment purchased with the portion of premiums attributable to term portion (b) insureds investment over the years plus the return that those investments have earned. HYPO: D, the insured, dies when the accumulated investment value of a policy (cash benefit) is $200K, and term protection is $100K. Under 101, the entire 300K is tax-free to the beneficiary. (Whatever the interest or other investment return included in the 200K is excludedthus life insurance can represent a considerable tax benefit).

9) Deductibility: No deduction shall be allowed for


(1) premiums on any life insurance policy (or annuity K) if the taxpayer is directly of indirectly a beneficiary under the policy or K.

29

10) Useful forms of life insurance a) Key Person Insurance: Buy a policy for teach of 4 principles in a partnership, and the business entity is the beneficiary. This provides breathing room for your business if key player dies. You cant deduct the premium, b/c business is beneficiary. After 9/11, some of the first life-insurance payouts went not to the victims families, but to their employers. (Unknown to most, corporations now are among the largest beneficiaries of life insurance , collecting on policies they purchase on the lives of employees For employers, the money they pore into the life insurance grows tax free, and gains on investments w/in the policies flow to the companies income statements each year. Those are tax free gains, unlike the kind the company would get from a stock portfolio. b) Buy-Sell Agreements: Company may buy out interest of dead guy w/out dipping into funds of a closely held corp, which usually needs all the money it has for the operation. c) Life Insurance Trust: M buys $1M insurance policy, transfers to life insurance (irrevocable) trust before prior to cash value growing to large number. This is taxable at this point, only to amount of cash value. Trust is beneficiary 1M is not used included in estate taxes, thus 1M, tax free, may be used to pay estate taxes. E. Charitable Contributions and Tax-Exempt Organizations: Any charitable contribution, as defined in 170(c), actually paid during the taxable year, is allowable as a deduction in computing taxable income irrespective of the method of accounting employed I.R.C. 170(c) a contribution or gift to or for the use of(basically) the government or any entity organized and operated predominantly for charitable, religious, scientific, literary, or educational purposes. *Contributions do not qualify under 170(c) merely b/c the recipient is tax-exempt, i.e., social welfare org., chamber of commerce, and social clubs are tax-exempt, but gifts to them are not deductible under 170 (although they may so be under 162business expense). *Gifts to needy individuals are not deductible under 170, even if a charitable organization plays a role in delivering the gift.

a. In General: To be deducted under 170 a charitable transfer must:


(1) Be paid to or for the use of a qualified recipient -the latter phrase encompasses gifts in trust -Expenditures made for the use of, but not to do not qualify for the 50% limitation of 170(b)(1)(A), but only for the 30% of 170(b)(1)(B). (2) Constitute a gift or contribution rather than a payment for goods or services -detached and disinterested generosity of Dubserstein? -This standard may be tough, as very few whom donate cloak themselves in anonymity to avoid the plaudits of the public gratitude

(i)

Under 170(l), if a payment to a college or university would be a deductible charitable gift but for the fact that it entitles the taxpayer to purchase seating at an athletic event in an athletic stadium of such institution, 80% of the payment

30

is considered a charitable contribution. The actual cost of the tickets is nondeductible. HYPO: T donated 2k to the University Athletic Fund (Donors of $1500 or more can purchase tickets which entitle the holder to preferred seats at basketball games) 80% of the $1500 is deductible, as $1500 entitles the taxpayer to the purchase. 100% of the $500 is deductible (3) Consist of Cash or qualified property; -You may deduct unreimbursed out of pocket expenses incurred in performing services for a charity (including uniforms, transportation, meals, lodging) which are considered gifts to the charity; but not time spent performing services for charitable organizations. (4) Not exceed a specified percent of taxpayers AGI in the year of payment -Most contributions: 50% -Contributions to private organization; gifts of appreciated property: 30% (5) Meet Certain other standards Taxpayer must itemize deduction under 63(e) to claim the deduction. b. Substantiation Requirements for All Contributions: 170(f)(8) disallows a deduction for any contribution of $250 or more, unless the T obtains substantiation of the contribution by means of written acknowledgement of by the donee organization.

(1) Limitation on Amount Deductible: The charitable contribution deduction for a payment of a taxpayer makes, partly in consideration for goods or services from the donee, must reduced by the fair market value of the goods or services the organization/donee provides. The taxpayer may rely upon donees representation of the FMV. EXCEPTION: Certain goods that have insubstantial value (small items of token value) are disregarded. -Annual membership benefits are ignored if the annual membership fee is not more than $75 and the benefits consist of only rights exercised in conjunction w/ being a member (free parking, discounted admission, preferred access to goods) (2) Value of a contribution in Property: If charitable contribution is made in property other than money, the amount of the contribution (which may be deducted) is the fair market value of the property at the time of the contributionbasis is disregarded. a. Substantiation Requirements: Most non-cash contributions raise valuation questionsvaluation being a matter of opinion. Treas. Reg. 1.170A-13(c) requires most taxpayers whom claim deductions in excess of $5K to obtain an appraisal and submit an appraisal summary w/ their returns. If under $5K, the donor must retain reliable written records w/ specified information.

HYPO: T plans to sell appreciated stock, and give the proceeds to a charity. T is dumb and would better off to contribute the stock itselfgiving rise to the same deduction as the donation of the proceeds, and the gift of the stock is not an event which requires T to recognize the gain (contrast 84).

31

(3) Contribution of Services: No deduction is allowable under 170 for a contribution of services. However, unreimbursed expenditures made incident to the rendition of services to an organization, contributions to which are deductible, may constitute a deductible contribution, i.e., the cost of a uniform which is required to be worn in performing donated services is deductible. (4) Percentage Limitations: (a) Individual (See 170(b)(1)In the case of an individual, any charitable contribution shall be allowed to the extent that the aggregate of such contributions does not exceed 50% of the taxpayers contribution base for the taxable year. (Individ. are entitled to deduct up to 50% of their AGI for most charitable gifts) HYPO: If AGI is 100k, T could deduct 50k of a 70k charitable contribution. 20k would be deductible the next year. (b) Corporation In the case of a corporation, the total deductions shall not exceed 10% of the taxpayers taxable income computed. See 170(b)(2). Haverly v. United States F: T, a Principal of Elementary School, received unsolicited samples of text books, with a FMV of $400. T could do whatever he wanted w/ the books, and chose to donate them to the Library. T then took a charitable deduction of $400, under 170. IRS assessed a deficiency against T representing income taxes on the value of the books received. T paid tax, now seeks refund. Dist. Court said that receipt of books did not constitute income H: Revd. When a tax deduction is taken for the donation of unsolicited samples the value of the samples received must be included in the taxpayers gross income. R: The receipt of textbooks is clearly an accession to wealth. The act of claiming a charitable deduction manifests the intent to accept the property as ones own. This is not a gift per Duberstein, the motive of the publishers was quite clear. Professor: Before he claimed the deduction it is technically still income per Glenshaw Glass, i.e., an accession to wealth. However, these simple accessions (receiving free toothpaste, books) are not included in gross income b/c of the time and aggravation it would cause the IRS. However, when you decide take advantage by claiming a charitable deduction, these small items must be included in gross income. (5) Contributions of Appreciated Property: The amount deductible is the FMV of appreciated property. Thus the T can deduct the present value of the property w/out realizing gain or loss they would otherwise incur if they sold the property and donated the proceeds. HYPO: 100k basis in stock, FMV is 80k. Should you give away or sell. Sell, take loss, and then contribute 80k. HYPO: T owns stock that cost her $2k, but is now worth $10k. If she gives it to charity, she can deduct the full 10kthe $8k gain going untaxed. However, note the following exceptions: (a) IRC 170(b)(1)(D): The amount of the deduction is limited to 30% of the donors AGI (instead of the regular 50%) (b) Unrelated tangible personal property: Where such a gift is unrelated to the charitys charitable purpose/function, the deduction is only your basis. See 170(e)(1)(B)(ii) HYPO: Painting worth 10k, basis is 7k, is donated to a medical research clinic the amount deducted must be reduced by the amount by which the value exceeds the donors basisonly 7k would be deductible. (c) 170(e) limits deduction to the propertys basis in the case of most contributions of ordinary-income property, short-term

32

capital gain property, and tangible personal property which the donee will not use in carrying out its exempt functions. F. State, Local and Foreign Taxes (1) Taxes deductible: 164 (a): Except as otherwise provided, the following taxes shall be allowed as a deduction for the taxable year w/in which paid or accrued: (1) State and local and foreign real property taxes (2) State and local personal property taxes (3) State and local and foreign income, war profits, and excess profits taxes. 164(d) Apportionment of taxes on real property b/t seller and purchaser -Clear allocation pro rata, i.e., if seller occupies for 100 days, he would pay 100/365, and purchaser would pay the rest. 164(f) Self-Employment Taxes -As a partner or self-employed individual, you must pay all of this tax. This says you may deduct portion (above the line) that employer would normally pay. G. Personal Exemptions and Filing Status: 1. Personal Exemption: 151 allows a deduction in a fixed dollar amount ($2,000 adjusted for inflation) for each taxpayer regardless of age, income, marital status; deducted from AGI in determining taxable incomein other words, below the line deduction This exemption amount is phased out (reduced) as the taxpayers AGI increases beyond specified levels, depending upon filing status. See 151(d) 151(d)(2) denies a deduction to a person (regardless of age) for whom a dependency exemption is allowable to another. When a joint return is filed, two personal exemptions are allowed b/c both spouses are taxpayers. 2. Dependency Exemption: allows a taxpayer to take a dependency exemption (deducting amount from AGI) for each individual whom: (a) is related to the taxpayer in a manner specified by 152, which is broadly defined and easy to be considered a dependent. (b) receives more than one half of his support (food, shelter, clothing, education, etc.) from the taxpayer or satisfies a special support requirement; and (c) does not have gross income exceeding the exemption amount or is a child of the taxpayer who is either below the age of 19 or a student under the age of 24 (See 151(c)(1)(A) General rule is that in order to claim dependency exemption, dependent may not have gross income over exemption amount; unless kid falls w/in 151(c)(1)(B) The taxpayers spouse can never be the taxpayers dependent. See 152(a)(9) If you are married and file a joint return w/ spouse, your parents could not claim you as a dependent. A. Divorced Parents: Who gets to claim the dependency exemption, where a kid gets over half of his support from his two parents? (1) Custodial Parent: Largely to simplify things, 152(e) was enacted, under which children are dependents of the custodial parent (the one who had custody for the greater portion of the year) unless the parents agree to transfer the exemption to the noncustodial parent. (2) Requirements: For 152(e) to apply, the following must be met: (a) more than one half of the childs support for the year is provided by his or her parents

33

(b) the parents are divorced, separated under a decree, or live apart for last 6mo. of the year. (c) the child is in custody of one or both of the parents for more than one half of the year. Thus 152(e) does not apply if a grandparent or other 3rd person supplies more than one half of the childs support or has custody for more than one half of the calendar year If a spouse remarries, support by a subsequent spouse of either parent is deemed to be provided by that parent. See 152(e)(5) (3) Transfer to Non-Custodial: Dependency status may be transferred to the noncustodial spouse: (a) Relinquishment: The custodial parent may relinquish the exemption by signing a written declaration that he will not claim the kid as a dependent. The declaration, when attached to the noncustodial parents return, entitles that parent to the dependency exemption. The parent providing 100% of the support could still do this. You would not want the high bracket parent to get the exemption b/c it might be phased out b/c of his/her high income.

B. Multiple Support Agreements: When two or more taxpayers contribute to another persons
support, 152(c) allows the group to determine which of them will be treated as satisfying the support requirement. This allows the members to have a free option to assign the dependency claim to the member who will derive the greatest tax benefit from it. (Person who gets exemption often is determined on a rotating basis) A multiple support agreement is allowed by 152(c) only if: (1) no one person contributed more than half of the beneficiarys support (2) more than half of the support is contributed by the parties to the agreement (3) each would be entitled to claim the beneficiary as a dependent if he or she alone contributed more than half of the support, (4) the taxpayer contributed over 10% of the support; and (5) each person contributing agrees not to claim the supportee as a dependent B. Gross Income Restriction: No dependency exemption is allowed for a dependent of the taxpayer who has gross income equal to or exceeding the inflation-adjusted exemption amount; unless the dependent is a child of the taxpayer who is under the age of 19 or is a full time student under the age of 24, or the dependent is permanently disabled This excludes the person, whom, though dependent on the taxpayer, provides a modest amount of self-support.

3. Filing Status: The taxpayers filing status determines the rates that apply to the taxpayers
income

a. Married Couples: If a couple is married at the end of the year, they can either
file separately or combine their income and deductions on a joint return under 6013. If one spouse has substantially higher income than the other, they can usually reduce their taxes by filing jointly. Filing a joint return makes both spouses liable for the tax. b. Single Persons: Singles must usually calculate their tax under 1(c), unless they are either a surviving spouse or head of household c. Surviving Spouse: Although single, the surviving spouse gets the benefits of the joint-return

34

Unmarried person whose spouse died during either of the two years immediately preceding the taxable year and who furnishes over half the cost of a household (in which the surviving spouse lives) that constitutes the principal place of abode for a child for which the person is entitled to a dependency exemption. d. Head of Household: A head of household is taxed more lightly than a single person, but not as lightly as a surviving spouse Unmarried person (not a surviving spouse) who furnishes over half the cost of maintaining a household (in which he lives) that constitutes a principal place of abode for one or more of the taxpayers descendents or any other person for whom the taxpayer is entitled to a dependency exemption. imperial H. Alimony and Related Issues: Until 1942, alimony was not income to the spouse who received it and was not deductible by the payor. 1) Current tax consequences: If divorced related payments to a spouse are alimony or separate maintenance (as defined in 71) the recipient includes the payments in income and (per 215, 62(a)(10)) the payor gets an above the line deduction, i.e., payor gets to deduct payment in computing AGI. 71 intentionally gives lawyers latitude to allow (family law) agreements to achieve favorable tax treatment. (In order to get deduction, someone must be taxed) -Compare Child Support and Division of Property, which does not invoke 71 (a) Child Support: Payor will pay 1k per month for as long as ex-lives. When kid is 21, payment reduces to $750. $250 thus represents child support not subject to 71thus the tax burden is on the payor. Why? It is not income to the payee, and the payor gets no deduction for making the payment.) (b) Division of Property: You get house, and 100K, I get benzo and vacation home. This is not alimony. Thus, by choosing to have payments come w/in or w/out the statutory definition of alimony or separate maintenance payments, the parties can decide on whom to throw the tax burden Payments which do not meet the definition of alimony are not deductible to the payor and not income to the payee. If amount is not treated as alimony, it is typically treated as a division of property, not w/in the scope of 71. Professor continuously noted that spouses could opt-out of 71. See 71(b)(1)(B), noting that the instrument may designate that otherwise taxable and deductible payments will not be taxable and deductible. Flexibility to the spouses in negotiating treatment. 2) Economics of Divorce: Usually the higher-income spouse makes payments to the lower-income spouse. What is usually desired by both parties therefore, is that the payments be deductible to the payor and income to the payee. (If the payments are deductible to the higher-bracket payor, that saves the estranged couple more in taxes than it costs to have the payments income to the lowerbracket payee.) This leads to more after tax income to divide. 3) Basic Requirements For Taxable and Deductible Alimony (a) Payment is received by, or on behalf of, a spouse under a divorce or separation instrument. 71(b)(1)(A) (b) Payment is in cash or equivalent.

35

(c) The divorce or separation instrument does not designate the payment as a non-alimony payment. 71(b)(1)(B) (d) The payment is not for child support or property division. (e) In the case of a decree or legal separation or of divorce, the parties are not members of the same household at the time the payment is made. 71(b)(1)(C) (f) There can be no liability to make the payment in cash or in property after the death of the payee-spouse. 71(b)(1)(D) (g) The spouses may not file a joint return

4) Basis in Alimony Property: See rule 1041, which essentially operates in the
same fashion as the basis for a gift (1015), i.e., recipient will take the basis of the transferor. Carry-over, not a stepped-up basis. 5) Qualified Domestic Relations Order (QDRO): When dealing with qualified pension benefits (including IRAs), it is necessary to draft and obtain court approval of a QDRO, which is necessary to permit he plan trustee to make distributions to former spouse and to shift income tax liability to them 6) Indirect Alimony (a) Insurance Premiums: The premium payments must benefit the spouse. They may benefit both spouses, but then the full amount of the payments would not able to be deducted. (b) Payments related to housing: What happens if the decree states that one spouse can occupy the family home and that the other spouse makes the mortgage payments? If H makes mortgage payments on a house owned by W, the payments may be alimony. You could not make payments on an asset that you own Often, both spouses have an ownership interest, and both may be liable on the mortgage. I. Personal Tax Credits (1) Credit for Household and Dependent-Care Services: 21 allows a credit for amounts paid for child care and household services if the taxpayer has at least one child under the age of 13 or an incapacitated dependent; and the amounts are paid in order to enable the taxpayer to be gainfully employed.

a) Status of taxpayer: To be entitled to credit under 21, the taxpayer must maintain a
household that includes one or more qualifying individuals (see above) The taxpayer maintains a household only if the taxpayer alone (or along w/ a spouse) furnishes over half of the costs of the household during the relevant period. See 21(e)(1) (these expenses do not include the cost of clothing, education, vacations, life insurance, but rather property taxes, mortgage interest, rent, utilities, food consumed)

b) Qualified individual: (See 21(b)(1))


(1) A dependent of the taxpayer under the age of 13 for whom the

36

taxpayer is entitled a dependency exemption under 151(c); (2) a dependent of the taxpayer who is physically or mentally incapable of caring for himself, whether or not the taxpayer is allowed a dependency exemption (3) the taxpayers spouse if physically or mentally incapable of caring for himself. This restricts the credit to taxpayers who live w/ a qualifying individual, thus maintaining a qualified individual in a separate place of abode does not qualify.

c) Employment related expenses: (See 21(b)(2))


21(b)(2)(A)indicates that the following expenses qualify: (i) expenses for household services (i.e., ordinary and usual services per T.R. 1.44A-1(2)), and (ii) expenses for the care of a qualifying individual (well-being and protection) (Such termcare shall not include any amount paid for services outside the taxpayers household at a camp where the qualifying individual stays overnight) *No summer camp; however, there is an exception for services outside the taxpayers household. See 21(b)(2)(B) *No transportation to some service provider *No educational expenses for any grade higher that first. *Dependent Care Center (facility providing care for more than six individuals) is OK if it meets the requirements of 21(b)(2)(C) & (D) Expenses must be incurred while taxpayer is gainfully employed or actively searching for gainful employment Employment may be within or outside the home and may consist of self-employment

d) Limitations on amount of Credit: (See 21(d)))


(6) Individual: For an unmarried person, employment related expenses may not exceed the taxpayers earned income (7) Married: For a married couple, employment related expenses may not exceed the earned income of whichever spouse has the lower income. There is a special rule for a spouse whom is a student or incapable of caring for himself--21(d)(2) treats such a spouse as having earned income of $250 (if there is on qualifying individual) or $500 (if there is 2+ qualifying ind.)

e) Computation of Credit:
A. The credit (applied to the employment-related expenses) is a percentage of the taxpayers employment-related expenses, subject to a dollar cap. (1) AGI of 15k or less: Credit percentage is 35% (2) AGI of 15k through 43k: The percentage is reduced by one percentage point for each 2k by which AGI exceeds 15k (3) AGI exceeding 43k: Credit percentage is 20% HYPO: A, has an AGI of $22,500. As percentage is 31% (35% less the 3 increments of 2k by which AGI exceeds 15k, and one additional point for the $1500) B. This credit percentage is then applied to the employment related expenses paid by the taxpayer during the year. However, the maximum amount of employment-related expenses which can be taken into account (and thus reduced by the credit percentage) are as follows:

37

(1) $3,000 if there is one qualifying individual (see 21(c)(1)) (2) $6,000 for 2 or more qualifying individuals (see 21(c)(2))

f) Allocation of Expenses: Where a portion of an expense is for household services or for


the care of a qualifying individual and a portion of such expense is for other purposes, a reasonable allocation must be made. Only the portion of the expense paid which is attributable to such household service/care is considered to be an employment related expense. See T.R. 1.44A-1(c)(6). g) Payments to Related Individuals: 21(e)(6) indicates that no credit shall be allowed under (a) for any amount paid by the taxpayer to an individual whom: (A) for the taxable year, a deduction under 151(c) is allowable either to the taxpayer or his spouse (B) who is a child of the taxpayer who has not attained the age of 19 at the close of the taxable year. (2) The Earned-Income Credit: 32 provides a refundable credit to taxpayers with relatively small amount of earned income (the credit is phased out as the taxpayers income increases). The primary purpose of this credit was to provide tax relief to the low income individuals with children and to improve incentives to work. a) Qualifying Children: The percentage (of the earned income credit) varies according to the number of qualifying children in the taxpayers household (i) No qualifying children: 7.65% (ii) One qualifying child: 34% (iii) Two or more qualifying children: 40% CHAPTER 4 BUSINESS AND INVESTMENT EXPENSES A. Depreciation and Related Matters 1. Depreciation, or Cost Recovery: In principle, depreciation refers to the systematic allocation of an assets cost over its useful life, i.e., the period over which the asset may reasonably be expected to be useful to the taxpayer in his business or investment activity. T.Reg. 1.167(a)-1(b). In order to produce $$ you must utilize/purchase assets. Depreciation deductions allow taxpayer to recover these costs over time. IRC 167 Depreciation: There shall be allowed as a depreciation deduction a reasonable allowance for the exhaustion, wear and tear (including a reasonable allowance for obsolescence) (1) of property used in the trade or business, or (2) of property held for the production of income

b.

Straight-Line Method: The cost (or other basis) of the asset (less its estimated salvage value) is deductible in equal amounts over its useful life. T.Reg 1.167(b)-1(a) Most real property must be depreciate w/ this method

38

HYPO: T purchased a 10k business asset w/ a salvage value of 2k at the end of its estimated useful life of 5yrs. Ts annual depreciation deduction would be $1600 ($8000/5)

c.

Declining-balance Method: Accelerated method of depreciation, producing deductions in excess of those allowable under the straight line method in the early years of the assets life (Thus, b/c of the time value of money, taxpayers will generally prefer to deduct depreciationand thus reduce taxessooner rather than later and will therefore prefer this method when possible)

(i)

Depreciation and Basis: 1016(a) (2(A) & (B) requires that proper adjustment to the basis of property be made for (among other things) the amount allowed as a depreciation if that amount reduced the taxpayers income. Essentially, every deduction you take is subtracted from your basis, and thus reduces your basis. When you sell the property, you are producing capital gain. (ii) Policy Issues: Depreciation deductions are necessary in order to measure taxpayers net incomes. However, Congress also uses them to encourage both particular activities and investment in depreciable property.

d.
one needs to know

MACRS: In order to compute depreciation deductions, (1) the propertys recovery period: These periods range from 3 to 39 years (2) the method of depreciation to be used: Property in the 3, 5, 7, and 10yr classes will ordinarily depreciated under a double-declining (200%) balance method, and switching to straight-line when that method would give a larger deduction Most real property is depreciated on straight line (3) the convention for determining when property is treated as having been placed in service. personal property is usually depreciated by using a half-year convention, which treats of all of the personal property placed in service during a year as having been placed in service in the middle of the year. Thus, the depreciation deduction for the first year is half of a full years deduction 168: This section contains the depreciation schedule for most tangible assets. Salvage value is not taken into account in this , so the entire basis of the property may be deducted if the property is held for the recovery period.

e.

Depreciation of Improvements: 168(i)(6) requires that depreciation on improvements of or additions to property be taken as if the property had been placed in service at the time of the improvement or addition.

39

HYPO: Residential rental real estate is assigned a recovery period of 27.5 years (most other buildings is 39 years). T has had 10 years of making depreciation deduction, 27.5 left to go, and then builds an addition to the bldg. The addition must be deducted over a period of 27.5 years. You may not add cost to the basis of original.

f.

Intangible Property: 197 deals w/ intangibles that diminish over time, i.e., patents, licenses, goodwill, permits, etc (168 deals w/ tangibles) deductible (amortized) over 15 yrs. Expensing in Lieu of Depreciation: The code (179) provides that you may elect to treat the cost of certain qualifying propertytangible personal depreciable property for use in the active conduct of a trade or businessas an expense (deducting it all completely, currently) as opposed to depreciation. Shall not exceed 25k and amount cannot exceed the taxpayers taxable income from the active trade or business This flies-in-the-face of the concept of gradual depreciation deductions WHY? Just to promote business, economic stimulus. Organizational Expenditures: The organization expenditures of a corporation (articles of incorporation, fees for by-law and minute prep.) may, at the election of the corp., be allowed as a deduction over 5 yrs. You may elect out of the 60month amortization. Why? You may plan to operate at a loss.

h.

g.

Crane v. Commissioner F: Mr. C transferred to Mrs. C property w/ FMV of $262k, and (coincidentally) subject to $262k mortgage. Mrs. C operated property for 7 yrs. collecting rent and taking depreciation deductions of $25,500, and now sells property subject to the non-recourse mortgage for $2500 cash. I: What is her gain? T argues $2500. To figure out, we must know: What is her basis? $262k (Basis is determined by the value of the physical propertynot equity undiminished by the mortgage.) It was appropriate to take depreciation deductions, which would cause the basis to decline by same amount deducted. 262k 25,500 = $236,500 basis What is amount realized? T received only $2500 cash, but she was relieved of the mortgage, which is thus part of the amount realized. (2500 + 262k= $264,500) H: Amount of a non-recourse liability is included in the amount realized on sale of the property. Thus if the mortgage was still $262k at the time of sale, Ms. Cranes realized amount was $262k + $2500. Her adjusted basis would be 262k less the $25,500 depreciation she took Gain: 28,000 (264,500 236,500) This allows you to milk your depreciation, sell property, then start over b/c mortgage is included.

B. Capital Expenditures: Some business or investment costs, called capital expenditures are not
deducted immediately (as in 162, 212); instead they are included in the basis of an asset (i.e., capitalized) and are normally recovered either through depreciation or by reducing gain when the taxpayer disposes of the asset.

40

-Capital expenditures may be thought of as the costs of acquiring or improving assets expected to last substantially beyond the end of the taxable year in which the costs were incurred. -Some capital expenditures cannot be recovered through depreciation. For example, the cost of land purchased for a business use, must be capitalized. But, b/c the land is not expected to suffer a decline in value b/c purchase and disposition, the land is not depreciable.

(1)

Capital Expenditure: Expenditures made for assets with useful life of more than one year. This includes any amount paid out for new buildings (acquisition or construction) or for permanent improvements or betterments made to increase the value of any property or estate. (2) Basis Principle: 263 reflects the basic principle that a capital expenditure may not be deducted from current income in the year that they are made. Such expenditures are capitalized. (3) Assets other than Inventory: (a) PLAINFIELD-UNION TEST Plainfield-Union v. Commn: H: Repair to water-line pipes (after river water wrecked havoc) was currently deductible R: An expenditure which returns property to the state it was in before the situation prompting an expenditure arose, and which does not make the relevant property more valuable, more useful, or longerlived, is usually deemed a deductible repair. On the other hand, a capital expenditure is generally considered to be a more permanent increment in the longevity, utility, or worth of the property. Any properly performed repair can be said to add value to an asset, however, the proper test is whether the expenditure materially enhances the value, use, expectancy, strength, or capacity as compared w/ the status of the asset prior to the condition necessitating the expenditure. Here the useful life was not increased, the capacity not enhanced. Just returned to the original position. The cost of keeping property in an ordinary efficient operating condition may be deducted as an expense. See T.Reg. 1.162-4. [If this would have been capitalized, they would have added cost to the basis of the pipes, depreciated over time] (i) Midland v. Empire: Oil seepage in meat packing plant, T lined walls and floors w/ concrete to restore normal operating condition. This was a deductible expense. Repairs did not increase the useful life or value when compared to the pre-seepage period. If they would have provided for seepage in construction, easily includable in the capital investment per 262 (ii) Regenstein v. Edwards: floor of 3rd story of landlords bldg sagged. Steel columns and steel crossbeams were installed in order to permanently cure the defect. Entire expenditure was for the sole purpose of restoring the property to its former condition and allowed the deduction. Professor disagrees: life expectancy was extended. (iii) Mt. Morris Drive-In Theatre Co: Construction of drive-in theatre caused rain water to drain onto neighbors land, T now constructs a drainage system. Tax Court held cost of drainage system to be a capital expenditure, b/c need for drain should have been foreseen when theatre was built. The fact that you did it later is irrelevant. Premised upon foreseeability; (iv) General Plan of Rehabilitation: An expenditure which would be a repair if it had been the only such expenditure

41

for the year in question may become part of a capital expenditure if the repair is made per a general plan of rehab. HYPO: Cost of a new roof cannot be treated as repairs on the theory that the replacement of a shingle is a repair, so the replacement of a roof is many repairs. (v) General Hypos of Capital Expenditures: #1: Bldg. has asbestos. Is removal deductible? It does not extend life. Does it increase value? Possibly. M says you may deductbut #2: Retainer to law firm. After year expires and no merger activity, it is deductible. In the year that merger went through, retainer must be capitalized. A successful merger produces increased benefits, synergies, etcthus, the retainer fee (in that particular year) are allocable to some time in the future, not only this year. #3: New company selling software, gets advertising co. to promote. Deductible? You expect income to be produced over a number of years, not only one year. But Commn say you may deduct. Commn v. Idaho Power: F: Idaho Power uses their trucks in trade/business, thus entitled to 162 (depreciation deduction) I: Since these trucks were used in process of constructing facilities, do they get to deduct depreciation over scheduled life of trucks or do they have to take same amount of depreciation and capitalize it, making it part of cost of bldg. (This case is not about whether they can take a depreciation deduction, but rather what do they do with it? Do they deduct 20% this year, or do they capitalize H: Construction-related depreciation on equipment not an expense to the taxpayer of its day to day business is appropriately recognized as a part of the taxpayers cost or investment in the capital asset. Expenses, whatever their character, must be capitalized if they are incurred in creating a capital asset. (Match expenses to the income that they generate? Is it a capital asset? If so, instead of deducting depreciation itemstrucksover 5 years, you are going to deduct them over 39yrs.) When the asset is used to further the taxpayers day to day business operations, the periods of benefit usually correlate w/ the production of income. Thus, to the extent that equipment is used in such operations, a current depreciation deduction is an appropriate offset to gross income currently produced. However, different principles are implicated when the consumption of the asset takes place in the construction of other assets that, in the future, will produce income themselves. In the latter, the cost represented by depreciation does not correlate w/ production of current income. Rather, the cost (although presently incurred) is related to the future.

(i)

Wages: The Idaho Power Court indicates that wages paid in the carrying on of a trade or business qualify as a deduction from gross income. 162(a)(1). BUT, when wages are paid in connection w/ the construction or acquisition of a capital asset, they must be capitalized and are then entitled to be amortized over the life of the capital asset so acquired. (ii) Office v. House Hypo: If you replaced the roofs on these two buildings, what arguments would you advance for each? House: You would argue that it was a capital expenditure, which you may add to your basis, which results in less taxable gain when you sell. Office: You would argue that this is a repairan ordinary necessary expense. But you will lose says Morrison (iii) Repair v. Replacement: At what point does it turn from a repair, which is deductible, to a replacement, which is capitalized? There is no bright line rule.

42

Would 1-5 shingles on a roof be considered a repair, while 15-25 is considered a replacement? (iv) Recurring v. Non-Recurring: Most of the ordinary expenses (per 162) are recurring and noncapital in nature, while most capital expenditures are extraordinary in the sense of non-recurring.

(v)

See Encyclopedia Britannica v. Commn: Where income is generated over a period of years the expenditures should be classified as capital. The work commissioned for E.B was intended to yield them income over a period of years. It would make no difference if E.B hired Stewart as a mere consultant to its editorial board. Similar to Idaho Power, E.Bs payment to Stewart for the manuscript of the Dictionary was unambiguously identified w/ a specific capital asset.

INDOPCO v. Commn I: Are professional expenses (1.2M) incurred by a target corporation in the course of a friendly takeover deductible business. Taxpayer argues that based on Lincoln Savings, the test for capital expenditures is whether an asset was created or enhanced; and no asset was created or enhanced. H: Capital Expenditure. (Commn now seeks to capitalize intangibles). R: While an expenditure that creates or enhances a separate and distinct asset should be capitalized, it does not follow that only expenditures that create or enhance separate and distinct assets are to be capitalized under 262. While the presence of a future benefit is not controlling, it is undeniably important in deciding if an expenditure is capital. The expenses of planning this merger created substantial future benefits for INDOPCO (access to buyers technology, synergy w/ buyers products, convenience of having one S/H rather than 3500). (Burden to show a right to deduction is on the T). Unlike Idaho Power, what is the asset that you attach these costs to? How will you be able to recover cost if there is readily ascertainable asset. How long does INDOPCO get to write off 1.2M? Good-will period? Is it just a part of your capital account added to basis? Practicality, this does not work out well. (4) Inventories: Inventory refers to goods purchased or manufactured by a firm and held for sale as part of the firms ordinary business operation.

C. Business, Investment and Personal Expenses:


Three general provisions (Sections 162, 167, and 212) allow business and investment expenses to be deducted: 162: Trade or Business Expenses: Deduction (usually above-the-line) is available for ordinary and necessary expenses paid or incurred during the taxable year in carrying on any trade or business. -trade or business was initially interpreted narrowly by Higgins v. Commn, which refused to hold that an individual who owned extensive investments, for which he incurred substantial annual expenses (salaries, rental, etc) was not engaged in a trade/business. -The legislative answer to this narrow approach was 212 which expanded what is deductible to the expenses incurred for the production of income -Commn v. Groetzinger: Trade or Business is a subjective determination. You do not have to sell goods, have a secretary, have a hanging sign, etcIn finding dog gambling (for this specific T) to be a trade or business, Court said that T must be involved in activity w/ continuity and regularity and primary purpose must be for income/profit. A hobby, sporadic activity, or an amusement diversion does not qualify.

43

(i)

Employee Business Expenses: Working as an employee is a trade or business, and so an employees business expenses are deductible under 162(a), i.e., Union dues and subscriptions to professional journals Most employee business expenses are itemized deductions and are also misc. itemized deductions for purposes of the 2% floor of 67. Some are above the line. Per 62(a)(2)(A), reimbursement or other expense allowance arrangement w/ employer paid or incurred in connection w/ performance of services as employee.

212: Expenses for Production of Income: In the case of an individual, there shall be allowed as a deduction all the ordinary and necessary expenses paid or incurred during the taxable year (1) For the production or collection of income; (2) For the management, conservation, or maintenance of property held for the production of income; or (3) in connection w/ the determination, collection, or refund of any tax (state or fed), i.e, fees relating to preparing tax returns, tax planning, legal advice relating to tax consequences of divorce, etc, etc This embraces an individual when he is a seeker of profit, by allowing him to deduct $ incurred in that search. 212 enlarges the type of category of income (not just trade or business) to which expenses are deductible. 262: Personal, living, and family expenses: Unless otherwise indicated, no deduction shall be allowed for personal, living, or family expenses. i.e., premiums paid on life insurance, insurance for house you live in, expenses maintaining your household (rent, water, utilities) Expenses incurred in traveling away from home (transportation, meals, lodging) are not deductible, unless they qualify under 162, 212 Costs of commuting to work are personal and thus not deductible. 263: Capital Expenditures: No deduction (as of right now)for any amount paid out for new bldgs, or for permanent improvements or betterments made to increase the value of any property or estate.

a) Trade Or Business: If an activity is a trade or business, depreciation on


assets used in the activity can be deducted under 167(a)(1); and other ordinary and necessary expenses are deductible under 162(a). Many business expenses (162 is the workhorse) are above-the line deductions b/c of 62(a)(1), and thus not subject to the limits of 67 (see below)

b) Production of Income: If property is held for the production of income,


167(a)(2) authorizes depreciation, and expenses of managing, conserving, and maintaining the property are deductible under 212(2). Costs incurred for the production or collection of income are deductible under 212(1). Expenses deductible under 212 are ordinarily miscellaneous itemized deductions. The (negative) impact of such a designation is that 67 allows deductions for misc. itemized deductions only to the extent that the total of these deductions exceeds 2% of an individuals AGI. Yeager v. Commn

44

F: Yeager is a big-time investor, and he has deducted interest expenses that he incurred by purchasing securities on margin debt I: 162 or 212 H: Y is an investor, not involved within a trade or business R: Y invested in these stocks long-term as opposed to short term, thus investment activity. A trader is someone whose profits are derived from the direct management of purchasing/selling Thus the following implications will attach to Yeager: (1) he is subject to the limitation on the deductibility of investment interest per 163(d) (which says your deduction cant exceed your net investment income) (2) This is a 212 deduction (not a 162 deduction) which is below the line and a miscellaneous itemized deduction which is subject to the 2% AGI or 167 (3) Subject to overall limits of 68 Kenseth v. Commissioner F: Firm deducted 40% for attorneys fees after T won an age discrimination suit against his former employer. H: This 40% is income, and you may deduct it per 212, but you are now subjected to the following: (a) 2% floor is triggered per 67 (b) Alternative Minimum Tax implications (c) Total limit of 68 Taxpayers argument: I assigned the money to the law firm. Court rejected this and indicated that an assignment is ineffective to shift tax liability. HYPO: M assigns 50% of income to his son. Tax liability is not shifted. (You could assign 10% to A, 15% to B, 25% to C, and so on, until there is no tax liability on your income.) 1. CONNECTING EXPENSES TO ACTIVITIES (a) The Role of Motive: United States v. Gilmore F: Gilmore owns 3 car dealerships, and is involved in a divorce. Wife is claiming that she should get share of property (which is stock in GM). I: Are Ts legal fees ($40k) deductible? T claims 212(2)in that he is trying to conserve and maintain his property by expensing these legal fees. H: Not deductible per 262 (personal expenses are not to be deducted) R: 1) 212 is subject to the same limitations and restrictions that are applicable to those allowable under 162namely 262 renders non-deductible personal or family expenses. 2) Characterization as business or personal must derive from the claim of origin test of the litigation cost (which are sought to be deducted). The litigation cost arose from the marital relationshipclearly personal or familiar relate.

(i)

Claim of the Origin Test: HYPO: Costs of defending a criminal prosecution arising out of business activities are deductible HYPO: W suing H in a divorce obtains court order restricting activity of corp. all of whose stock is owned by H. The corps costs of seeking a relaxation or clarification of the order have both a business origin in the sense that they are incurred b/c of business necessity, but yet they have a personal origin (262) in that the dispute arose b/c of the divorce. Court allowed deduction under 162. HYPO: Lawyer retires, buys a boat which is his only asset. A client locates the lawyer and sues him for malpractice. Origin of the claim is business not personal

45

(ii)

Capital Expenditure Aspects: The SC has used an origin of the claim test to require capitalization of the costs of litigation arising out of the acquisition of an asset, which per T.R. 1.262(a)2 is a capital expenditure. Capital Expenditures: Cost of goodwill in connection w/ acquisition of an asset. Cost Acquisition, construction or erection of buildings, machinery, and equipment, furniture and fixtures, and similar property having a useful life substantially beyond the taxable year. (GILMORE IIallowed the taxpayer to include his legal fees in the basis of the property he kept his wife from getting) See page 314

Wild v. Commn F: T gets alimony, which is taxable per 71. T expends 6k in legal fees to secure alimony payments. I: Are these fees deductible under 212(1) or prevented by 212 H: Deductible under 212(1). R: T.R. 1.262-1 indicates that attorney fees which are paid in connection w/ a divorce or decree for support, which are properly attributable to the production or collection of amounts includible in gross income under 71 are deductible by the wife under 212. Gilmore (claim of the origin) interprets 212(2), this is a different section212(1). IRS has not changed the reg. so it stands. Pevsner v. Commn: (5th Circuit) F: Employer reqd T to wear YSL clothes to work and to some work related conferences. T could (if she wanted) also wear the YSL clothes outside of work, but she claimed that she did not and would not b/c it conflicted w/ her simple life style and would never have purchased this ordinarily I: May T deduct cost of clothing and upkeep of clothing per 162 or does 262 prevent? Tax Court: Yes, deductT would not herself have bought this stuff but for work, and she was not wearing it away from work. (SUBJECTIVE APPROACH) H: Revd. No deduction, as this is not a business expense. Test for deductibility of clothing is if it is of a type specifically required as a condition of employment and whether it is not adaptable to general usage as ordinary clothing. OBJECTIVE APPROACH must be utilized (individualized approach of tax court is too difficult), and this stuff is adaptable to general usage as ordinary clothing. Inherently Personal Expenses: Army mans cost of required haircuts over what he would have normally spent were inherently personal in nature and thus non-deductible. HYPO: M is professional musician. This year he spent $1k for formal clothing, which he wore only during concert appearances. Deductible per the Objective test. HYPO: Paintersbibs are not deductible HYPO: Steel Workers: steel toes and goggles are deductible. (b) Ordinary and Necessary Welch v. Helvering: F: Company (in grain business) goes bankrupt and is discharged from all debts. T was the secretary of the Co. when it went broke. T now owns his own grain business, and cuts a deal w/ Kellogg to purchase grain on commission. T (voluntarily) pays off the debts of his old company to improve his chances of success and re-establish relations. I: Are Ts payments to the creditors deductible as ordinary and necessary business expenses per 162? H: Not deductible. Capitalize and add to your basis, as this is going to produce good will income over a long period of time (1) Necessary? Yes. Necessary for the development of the business (i)

46

(2) Ordinary? No. Voluntariness of the actions are irrelevant, this simply is not ordinarily donein factit is extraordinary. Court will not expand what is ordinary.

(i)

Deputy v. Dupont: An expense may be Ordinary though it happens but once in the Taxpayers lifetime, if the transaction which gives rise to it is a common or frequent occurrence in the type of business involved. Thus, the same expense may be ordinary for one business, while not necessarily so for another business. Normalcy in the particular business/industry is crucial and controlling.

2. A REASONABLE ALLOWANCE FOR SALARIES (a) Salary v. Dividend: One cost of doing business that is not deductible is a corporations payment of dividends to S/H. Since dividends and salaries are ordinary income to the recipient, but dividends are not deductible by the corporation, the owners of a closely held corp have every incentive to pay high salaries and low dividends, or no dividends at all. 162(a): Trade or Business Expenses: (1) A reasonable allowance for salaries or other compensation for personal services actually rendered. Per T.R. 1.162-7(a): The amount of compensation must be reasonable and the payments must in fact be purely for services. -any amount paid in the form of compensation but not in fact for services, is not deductible. -Bonuses to employees will constitute allowable deductions from gross income when made in good faith as additional compensation for services actually renderedprovided such payments (when added to fixed salary) do not exceed reasonableness. Elliotts, Inc. v. Commissioner F: T, company in the business of selling John Deere equipment, claimed deductions of $181k and $191k in consecutive years for the total compensation to its CEO and sole shareholder, Edward G. Elliott. The Commn found these to be in excess of the amount T properly could deduct as reasonable salary under 162(a)(1). Tax Court agreed, saying that 120k and 125k were reasonable salary, and assessed deficiencies. I: Deductibility by corporation of payments made as compensation for services to an employee who is also a S/H. Salary (deductible per 162) vs. dividends (not deductible) H: TC is revd. R: 1) Automatic Dividend Rule (presuming disguised dividends if corp is profitable) is rejectedas no statute requires profitable corporations to pay dividends and not all s/h demand dividends. 2) Court focuses on reasonableness of compensation when there is no other evidence of an intent to hide dividends into compensation payments. REASONABLENESS: Nature and Quality of services; scarcity of qualified employees Effect of those services on investment returns Look at other people in circumstances How important is individual M wants to know why the marketplace doesnt just decide what is reasonable Tax Court failed to consider Elliotts extreme personal dedication and devotion to work, i.e., 80 hours per week. TC failed to consider the reasonableness on an overall basis.

47

(i)

Automatic Dividend Rule: Although Elliotts expressly rejected this rule, it has resurfaced in the 9th Cir. and the IRS indicates that failure to pay more than an insubstantial portion of earnings as dividends on stock is a very significant factor in determining the deductibility of compensation

HYPO: Ms daughter doesnt do anything to justify her salary of 30k. 1) If not reasonable compensationdoes she still pay tax. YES (Probably). Some may say gift 2) M (sole s/h), Commn would say that this is a constructive dividend (to you) described as salary. HYPO: M rents bldg. to his Co. for 50k per year. FMV is 25k per year. RESULT: Rental payments are deductible, but Co. is likely to be denied 25k (over FMV) in deduction. M still has income. 3. TRAVEL AND ENTERTAINMENT: The issue that is involved here is paying for something and trying to deduct it, although it is really 262 (personal stuff). Are these expenses brought upon by business or personal? Because these expenses have significant personal expense elements, the decisions try to insure that only expenses that are predominantly business in nature are deductible. *SEE HANDOUT 3/24, 3/25

(a)

Commuting and Lunching Expenses (1) Commuting: Basic rule is that commuting expenses are not considered business expenses, and thus, they are not deductible. (See Commn v. Flowers, holding that lawyer who lived in Jackson, MI could not deduct costs of traveling to Mobile, AL) Exceptions: -When additional expenses are incurred for transporting job required tools and an allocation b/t personal and business expense is feasible. -Taxpayer whom is on duty while Commuting: Pollei v. Commn allowed police captains to deduct costs of commuting to work in their personally owned unmarked cars, a regularly scheduled activity.. -Costs of commuting to a distant, temporary job are deductible as travel expenses -Costs of travel b/t one job site and another are not commuting expenses and thus deductible (as when a lawyer takes a cab from the office to the courthouse) If you go away overnight, you may deducttravel, lodging, meals. REVENUE RULING 99-7 I: Under what circumstances are daily transportation expenses incurred by a T in going b/t the Ts residence and a work location deductible under 162(a). H: In general daily transportation expenses incurred in going b/t a Ts residence and a work location are non-deductible commuting expenses. However, such expenses are deductible under the following circumstances: (1) T may deduct daily transportation expenses incurred in going b/t the Ts residence and a temporary work locale outside the metropolitan area where the T lives and normally works (2) If T has one or more regular work locations ways from his residence, the T may deduct daily transportation expenses incurred in going b/t the Ts residence and a temporary work location in the same trade/business, regardless of distance. (3) If a Ts residence is his principle place of business (exclusively used on a regular basis) the T may deduct daily transportation expenses incurred in going b/t the residence and another work location in the same trade/business, regardless of whether the other work location is regular or temporary and regardless of distance.

48

(b) Working Couples: Suppose that H&W work in cities 300 miles apartheaving commuting expenses for at least one. It seems artificial to say that the expenses derive from a personal choice about where to liveyet not exception for this kind of case has yet been created. Moss v. Commn F: T, lawyer, appeals decision of the TC disallowing deductions of a little more than 1k in two years, representing his share of his law firms lunch expense. There is no suggestion that the lawyers dawdled over lunch, as they were extremely busy w/ much litigation. The IRS concedes that meals are deductible under 162 when an ordinary and necessary business expense. H: No deductiondaily lunches is too often R: 162 is not in play. (T is not an employee, but rather a partnerself-employed.) Court assumed that it was necessary for Mosss firm to meet daily to coordinate work of the firm (and lunch was the most convenient time) But it does not follow that the expense of the lunch was a necessary business expense. The restaurant they went to was not luxurious, it may be different if they went to a place that was too expensive for their personal tastes, so they would have gotten less value than the cash equivalent. To allow a deduction for all business-related meals would confer a windfall on people who can arrange their work schedules so they do some of their work at lunch. The difference in this case is that all the participants in the lunch knew each otherthey dont need the social lubrication that a meal w/ an outsider providesat least dont need it daily. Matter of degree: If they met once a month, they could probably deduct. But they tried to take advantage of the system and do it too much. Matter of degree and circumstancedaily is too often. (i) If T (Moss) was an employee, rather than a self-employed taxpayer (partner) the issue would have been whether the cost of the meals was includible in gross income When one T pays for a business meal which another T eats, issues of both deductibility and includability arise: The person who eats the meal (an employee, for example) needs to determine if the meal can be excluded under 119 or as a de minimis fringe benefit; the person who furnishes (employer) will usually treat the cost of the meal as a business expense under 162(a). If the lunch was held in the conference room, catered by Caf Angelo, this would be deductible per 119 (meals are furnished on premises) See 274, you may only deduct 50% of the cost of meals.

(ii) (iii)

c. Away from Home: 162(a)(2) allows a deduction for travel, meals and lodging while away
from home in pursuit of a trade or business. -The IRS has reasoned that the Ts home for purposes of 162(a)(2) is the Ts principal place of business HYPO: T, on a 3-day business trip to another city, can deduct meals and lodging. HYPO: A T who takes a permanent job 500 mi. from home cannot deduct meals and lodging at the job site even if this is away from home in a literal sense. Robertson v. Commn F: Justice Robertson is a judge in Jackson, MS and lives and teaches law in Oxford, MS. Judge got reimbursed for some of his travel, meals and lodging incurred in attending court sessions in Jackson, and tried to deduct what was not reimbursed, saying that he was away from home. H: No deduction. Tax home is Jackson, as his trade/business is being on the Supreme Court. Thus the traveling expenses were not incurred while away from home

49

Hantzis v. Commn F: T goes to law school in Boston, lives in Bostonworks in NYC for summer. T seeks to deduct travel expenses, apartment that was rented, and meals under 162. H: Not deductible. R: Ts home for purposes of 162(a)(2) was her place of employment and the cost of traveling to and living in NY was not incurred while away from home. Court will ultimately assess the reason for the Ts maintenance of two homes. If the reason is perceived to be personal, the Ts home will generally be held to be his place of employment, rather than his residence, and the deduction will be denied. Ts trade or business did not require that she maintain a home in Boston as well as in NYC. (Compare situation where T lives and works for a firm in Boston, and business exigency requires that she temporarily live in NYC to serve a client) (i) Temporary Business: You may deduct expenses if away from home on a temporary basis 162(a)(2) provides that the taxpayer shall not be treated as being temporarily away form home during any period of employment if such period exceeds one year.

HYPO: Visiting law professor for 9 months (school year) may deduct lodging meals, travel, etc HYPO: If the dean asks the visiting professor to stay on for another yearthe expenses are deductible until the time you make the decision to extend your stay past one year. HYPO: If the visiting professor had signed on for 18mo., but then quit after 9mo., none of this is deductibleb/c the expectation was that the employment was to last for more than one year. d. The carrying on requirement (1) Start up and expansion costs: Some costscalled pre-opening expensesare nondeductible under 162 b/c no business is being carried on. These non-capital business start up expenses may be written of over a five year period, if the trade or business is an active one. (2) Costs of seeking employment: It is now settled that an employee who incurs expenses in seeking a new job in the same field can deduct those expenses under 162. -However, a law student who flys to CA to look for a job does not get deduction. (3) Moving Expenses: 217 allows an above the line deduction for some of the costs of moving to take on a new job position (over 50mi. away) even in the case of an employee or selfemployed person who had no job before the move. -lodging, mileage, Uhaul. BUT not meals

D. Public Policy Limits: No deductions based upon Public Policy


Tank Trunk Rentals: Co was fined for overloading trucksand sought to deduct fines as a cost of business. See 162(c)(f) See handout 3/26 See problems page 395 See handout 3/19 CHAPTER 5 INTEREST INCOME AND DEDUCTIONS

50

A. Forms of Interest: Interest is includable in the lenders income under 61(a)(4) unless 103 applies. 163 seems to say that all interest can be deducted, however, several Code provisions limit or deny altogether the deductibility of many kinds of interest. -The simples form of interest is an explicit annual charge of a percentage loan for the period in which the loan remains outstanding. But other ways of charging are common: (i) Points: Banks often charge loan processing fees or points to borrowers. The IRS treats points as interest unless they are paid for such services as preparing and reviewing documents, doing the paper work required for FHA and VA mortgage loans, and appraising the property that secures the loan. -Most points on loans (other than home-mortgage loans) cannot be deducted when paid, but rather must be written off over the term of the loan under 461(g).

(ii) (iii)

Carrying Charges: 163(b) contains a formula for allocating carrying charges on installment-purchase contracts b/t interest and other payments Imputed Interest: Under 483 and 1274. Seller and buyers must treat part of the deferred purchase price on most sales as interest if the agreement does not provide for stated interest at an adequate rate.

B. Deductible and Non-Deductible Interest: See handout 3/27 -Interest: The cost of obtaining current cash, borrowing (and incurring interest) to buy a particular asset -163: There shall be allowed as a deduction all interest paid or accrued within the taxable year on indebtedness. But this general rule is limited by a number of Code provisions which deny deductibility to many kinds of interest: EXCEPTIONS TO GENERAL RULE OF DEDUCTIBILITY: (1) Interest on indebtedness incurred or continued to purchase or carry tax exempt bonds cannot be deducted; 265(a)(2) (2) Passive Activity: Interest which is treated as an expense of passive activity (such as renting property) is deductible only against income from passive activities if the taxpayer is an individual; 469 (3) Interest on Indebtedness: incurred or continued to purchase or carry certain lifeinsurance and annuity Ks cannot be deducted; 264 (4) Investment Interest: As defined in 163(d)(3), can be deducted by a non-corporate taxpayer only to the extent of the taxpayers net investment income for the year. (5) Construction of Real Property or other Property with a long useful life: Must be capitalized if the property has an estimated period of more than two years or if the estimated production is more than one year and the property costs more than $1M; 263A(f) (6) Personal Interest: Personal interest incurred by a non-corporate taxpayer is not deductible. 163(h) defines personal interest as all interest other than interest listed in 163(h)(2)(A)(F)

1. Personal Interest: -In the case of a taxpayer other than a corporation, 163(h)(1) disallows deductions for personal interest, which is defined as all interest other than the six kinds of interest listed in 163(h)(2), i.e., (1) interest paid or accrued on indebtedness properly allocable to a trade or business (T has a store, borrowing $ for inventory, etc) (2) any investment interest (but only to the extent of the incomeYeager) 51

(3) any interest taken into account under 469 in computing income or loss from a passive activity of taxpayer (4) any qualified residence interest (most home-mortgage interest) -consists of interest on acquisition indebtedness and on home equity indebtedness. (5) any interest allowable as a deduction under 221 (relating to interest on educational loans) (these are above the line per 221. Unless the T can find refuge in one of these safe-harbors, the interest, such as consumer interest, credit card interest, interest on auto loans, and the like will not be deductible. 2. Tracing under 163: Temporary (Proposed) Treas. Reg. 1.163-8T prescribes tracing rules for 163(d) & (h). -Under the regulationsdebt is ordinarily assigned to activities on a flow-of-funds version of tracing, i.e., someone who borrows 100k, using the borrowed money to pay for business expenses and then uses 100k of previously accumulated cash to buy a personaluse airplane may deduct the interest on the 100k debt. CHAPTER 7 Who is the Taxpayer? The code is almost silent on the subject of choosing the person to be taxed as a result of incomeproducing events. PRINCIPLES: (1) If you are assigning future income from personal servicesnot goodyou must still pay tax (Lucas v. Earl) (2) Assignments of fruit only are ineffective, when T retains the tree (Horst) A. Personal Service Income Lucas v. Earl F: Husband and Wife enter into a K in 1901 which stipulates that all property they get (salary, gifts, inheritance, etc) is to be held in joint tenancy with right of survivorship. I: Should husband be taxed for the whole of the salary and attorneys fees received by him in 1920 & 1921, or should he be taxed only half of that, b/c of the arrangement w/ his wife. H: Although this is a valid agreement, H is taxed on the whole R: H earned the money and a gratuitous assignment will not be effective, regardless if the K is valid. Income tax cannot be escaped by anticipatory arrangements and contracts however skillfully devised to prevent the salary when paid from vesting even for a second in the man who earned it. Salary for H: 24,000 (4,000 x 6%) = (20,000 x 12%) = 240 2400 2640

H says that b/c of agreement it should look like this: 12,000 (4,000 x 6%) = 240 (8,000 x 12$) = 960 1200 x 2 = 2400

52

HYPO: M&J have a partnership. Profits are 300k. 250k is attributable to Js hard work. 50k to Ms occasional work (done on golf course). You cannot assign incomethus metaphor does not always work. This inequity should be addressed by the Partnership. HYPO: Law Professor works in Clinic, and is sometimes entitled to $, which is turned over to the school. Income to the Professor? NO. Acting as an agent of the organization. (School will probably not pay tax as it is a tax-free institution.) HYPO: W is named in Hs will as executor, and per law is entitled to a fee. W waives the fee, to avoid income. NOT TAXABLE. HYPO: M says to BOD, I dont want any more salary (1.2M). BOD gives it to UCLA, to establish research fund. M not taxed, as he had nothing to do where it went. (If he suggested that it go to UCLA, then it is different.) GIANNINI CASE See Problems page 437. Poe v. Seaborn H: Husband and Wife are entitled to file separate returns, each treating one-half of the community income as his or her respective incomes. Under Washington (State) law, the entire property and income of the community can no more be said to be that of the husband, than it could rightly be termed that of the wife. Earl was distinguished, b/c here, by law, the earnings are never the property of the husband, but that of the community. This is income derived from investing selling community property (not the salary of the husband). Ownership counts, not control B. Income From Property Blair v. Commissioner F: Father (life income beneficiary of his fathers will) assigned to his child fixed annual dollar amounts of income to be paid from his life interest in a trust. I: Who is taxed? Father or Sons {Commn argues that the income was his, and the assignment was merely a direction to pay over to others what was due to himselfper Earl} H: Valid assignment of property, and the child, rather than the father are taxed on the assigned income as it is received. Earl is not applicable, but rather Poe is b/c the income is arising from the property. R: The one who is to receive the income as the owner of the beneficial interest is to pay the tax. The interest that the father had was present property alienable like any other in the absence of a valid restraint upon alienation. The beneficiary (father may thus transfer a part of his interest as well as the whole. -Fathers life interest in the trust income is the treethe underlying property. The father in effect chopped down some of his fruit and some of his tree and gave some of his fruit and some of his tree to his children. This was effective to shift tax liability. If you only give away the fruitthis is an assignment of income, which will not shift tax liability. Helvering v. Horst F: (Coupons were attached to bonds, when date arrived, you clipped and turned into bank) Owner of negotiable bonds detached from them negotiable interest coupons shortly before their due date and delivered them as a gift to his son who in the same year collected them at maturity. I: Is the realization of income taxable to the donor--father. H: Yes. The exercise of the power to dispose of income (procuring payment to another) is the enjoyment and hence realization of the income. R: The enjoyment of the economic benefit accruing to donor by virtue of his acquisition of the coupons is realized as completely as it would have been if he had collected the interest in dollars.

53

This is not like Blair v. Commn. In Blair, the right to the income from the trust property was derived from a gift of ownership of the income producing property. (Since the gift was held to be the income of the owner of the property the income from it was held to be the owner of the property) Here, the donor (owner) is retaining control of the property and thus the income is taxable to him although paid to the donee. The donor had given only the fruitthe couponrather than the treea bond, thus this is a CarvedOut Income Interest. If he had devised bond + interest, this looks like Blair (tree and fruit) TEST: Identify the treethe property that is helddoes the donor still have rights to the tree, or has he parted w/ some of the tree? (You dont need to part completely w/ the tree) D: Through the gift, the coupons became the absolute property of the donee, free from the donors control and in no way dependent upon the ownership of the bonds. See problems page 464

C. Taxation of the Income of Minor Children


1(g) indicates that in the case of a child under the age of 14, income in excess of 1,000 is generally taxed at the parents marginal tax rate. In addition, 151(d)(2) denies a personal-exemption deduction to a taxpayer (regardless of age) for whom a dependency exemption is allowable to another FAMILY PARTNERSHIPgiving away interest to children 1) Apartment bldgDad owns and gives away interest in capital partnership to 2 kids. Each getting a 10% chunk of income. 2) Accountant: This will not work as this would just be shifting income (fruit) and not the tree. Looks like Lucas 3) Lawyer: Building that Dad practices in has depreciated almost down to $0. Dad gives to kid, and pays 1000k rent, thus creating a 162 expense, thus a deduction. IRS: If they (kids) dont do anything to earn the $, allocate salary to a reasonable Partner-, i.e., the father. Chapter 8: Income From Dealings in Property Eisner v. Macomber held that the 16th Amendment did not authorize the taxing of unrealized gains (few today would believe Macomber is good law today). Still today, Congress has generally chosen not to take unrealized gain into account b/c of the administrative and compliance difficulties that would result. HYPO: Purchase stock for $40, at the end of the year it is worth $100. Wealth is increased by $60, but T does not include the increment in income b/c the stock has not been disposed of, i.e., not realized.

(1) Analysis of a Property Transaction:


(a) Has a sale, exchange, or other taxable event (realization event) occurred? (b) If so, determine the amount of realized gain or loss ( 1001(a)) by comparing the amount realized ( 1001(b)) with the adjusted basis ( 1011(b)) of the property disposed of. (c) Even if realized, should this gain (or loss) be recognized (deductible)? Although 1001(c) generally requires that realized gains/losses be recognized, Congress has provided for non-recognition. -(Recognized Loss is deductible only to the extent that it is allowable. 165(a) generally allowed deductions for losses sustained during the year, but 165(c) prohibits non-corporate taxpayers from deducting losses other than those

54

(1) incurred in trade or business, (2) those incurred in a transaction entered into for profit, and (3) certain casualty losses w/ respect to personal-use property)

(d) If it should be recognized (hence includable in gross income if a gain, or deductible if a


loss), you must determine the character of the gain or loss as ordinary or capital. If capital it may need to be treated specially in making the computation. A. The Taxable Event: A Gift of Property is non-taxable Bequests are non-taxable Exchanges of similar property are not taxed HYPO: M goes to get a $15 haircut at the barber. Pays with stock which has a FMV of $15, basis of $5. RESULT: M has $10 gain, as there has been a realizing event. (It is as if he went to a broker and cashed stock in, received $15, and gave it to the barber). The barber has $15 of income; basis of $15. International Freighting Corp., Inc. v. Commn F: Bonuses were given in the form of common stock of the duPont Company. During 1936, T paid over 150 shares of common stock of duPont, whose cost to T at the date of delivery was $16k, and whose market value was then $24k. T took a deduction for $24k in its return. Commn argues that if T took 24k deduction, it should have included 8k as taxable profit (realized gain) H: Taxable gain equal to the difference b/t the cost of the shares and the current market value, although they get a deduction per 162. Employees are taxed on the property received for services, per 83 R: 1) Tax Court properly held that market value at time of delivery was properly deductible as an ordinary expense of the business under 162 (as payment depleted Ts assets in an amount equal to the market value) 2) Tax Court properly held that transaction resulted in taxable gain to the T. First, this was not a gift, as the value of the shares would not then have been a deductible expense under 162. But rather delivery of the shares here constituted a disposition for valid consideration, and resulted in a closed transaction w/ a consequent realized gain It is irrelevant that T was under no legal obligation to award these shares or pay additional compensation to employees. The consideration received by the T must have been deemed to be equal (at least) to the current market value of the shares, or else the bonus would be invalid. *True, 1001(b) indicates that the amount realized is the sum of any money received plus fair market value of the property received. However, where there is a disposition of stock for services, no property or money is received by the person who disposes of the stockbut moneys worth is received and that and such receipt comes w/in 1001(b) See problems page 496 HYPO: T, cash-method, owes an employee, E, $10k for services rendered in Ts business. T satisfies the obligation by transferring to E property w/ an adjusted basis of $4k, and a FMV of $10k. RESULT: T has 6k of capital gain, and a 10k deduction per 162. E has 10k gain/basis (i) Note: T.R. 1.165-4(a) No deduction shall be allowed under 165(a) solely on account of a decline in the value of stock owned by the T, when the decline is due to a fluctuation in the market price of the stock.

B. Computation of Realized Gain or Loss 1. Basis of Property Acquired by Purchase/Taxable Exchange

55

(a) Basis of Property acquired by exercising an option: Suppose T (in 2001)


pays 10k for an option to buy business real estate for $100k, within the next two years. (When owner gets 10k option, she does NOT have income at this point) Currently, property is worth $95k. In 2003, T exercises option when property is worth $115k. RESULT: T recognizes no gain, and obtains a basis of $110k (10k option + 100k purchase price) (Owner would now have income of $110k) If the option expires with no deal going throughthe owner now has 10k of ordinary income. (Professor questions why basis isnt simply just reduced) If 3rd party wants to buy Ts option, and T sells it for $15k, T has $5k of gain income. (But is it ordinary or capital income?)

(b) Allocation of Basis: Consider a T who buys a 200 acre farm for 100k, and
later sells 50 acres for 40k, retaining the rest of the farm. The Ts amount realized is 40k, but what is the basis of the property sold? Gain realized on sale or exchange of property is included in income--generally gain is the excess of the amount realized over the unrecovered cost or other basis for the property sold HYPO: B purchases for 25k property consisting of a used car lot (10k) and adjoining filling station (15k). Five years later, B sells the filling station for 20k, at a time when 2k has been allowed as depreciation thereon. RESULT: Bs gain on the sale is 7k, as basis in the filling station was reduced to 13k via depreciation, sold for 20k. Inaja v. Commn F: Inaja paid $61k for recreational property in L.A. L.A. pollutes river, and pays Inaja $49k for easement to continue polluting. H: No Gainno way to figure out how to allocate your basis in property right (or part of property) that Inaja gave away, thus the 49K was treated as a tax-free return of capital. Inaja will now have a basis in the property of $12k ($61k $49k). HYPO: If county had paid Inaja $70k, basis still being $61, Inaja would have had $9k of capital gain. Basis would be $0, as you cannot have a negative basis. See problems page 502 2. Transactions involving Mortgaged Property Crane F: Crane took deductions on mortgage (260,000), and says that this was not basis I: When property is acquired, what is your basis? This issue is important, b/c this is where depreciation deductions come from. H: Mortgage debt is included in basis (Non recourse liability incurred on acquisition of property is included in basis), thus the amount of non-recourse liability is included in the amount realized on the sale. Parker v. Delaney F: Parker acquired 4 apartment houses from banks. He paid no money for the properties and took them subject to mortgage liens totaling $273k. In 1945 the mortgages were in default, and Parker conveyed the properties to the banks. Parker reported 31k as gain from the sale, but sued for a refund, claiming that his amount realized was zero. The adjusted basis in 1945 was $273k less $45,280 (of depreciation deductions), or $227,719. I: Was gain realized (in excess of the adjusted basis) from this disposition?

56

H: R: P purchased the property for $0. He paid $13,989 on mortgage debt, and took depreciation deductions in the amount of 45,280. 45,280-13,989 = 31,291, which is his gain. This is good timing for P, as he is taking deductions early, and taxed later. The amount of the mortgages was $31k in excess of the adjusted basis. When a mortgage is used to finance the purchase of property, that mortgage goes into the basis of the property. Then on subsequent foreclosure on the property (treated as a sale) the mortgage goes into the amount realized. Thus the gain is calculated by the amount realized minus the basis (which will be reduced by an depreciation taken) Woodsam v. Commn F: T buys blackacre for $500k cash and in a little amount of time, the FMV becomes $1M. T goes to a bank and says loan me $750k (non-recourse), secured by the property. T bails when FMV is 600k. H: 250k of realized gain. Bank gets property. R: When you turn property over, you realize indebtedness ($250k) BASIS ANALYSIS: If T takes the 750k to develop the property, his basis goes up. However, if he takes the money and goes gambling, basis is unaffected. INCOME ANALYSIS: No Income, as this is borrowed money. If value of property falls to 600k, still no income or change to basis. Revenue Ruling 90-16 F: X was the owner and developer of a residential subdivision, obtaining a loan from an unrelated bank, unconditionally liable for repayment of the debt. The debt was secured by a mortgage on the subdivision. X became insolvent and defaulted. An agreement was reached whereby the subdivision was transferred to the band and the bank released X from all liability for the amounts due on the debt. -FMV: $10k -Xs adjusted basis: $8k -Amount due on debt: $12k I: Is this transfer (to the bank) a sale or disposition resulting in the realization and recognition of gain by the T under 1001(c) and 61(a)(3)? H: R: To the extent of the FMV of the property transferred to the creditor, the transfer of the subdivision is treated as a sale or disposition upon which gain is recognized under 1001(c), thus X realizes and recognizes $2k gain. Subject to the application of 108, to the extent that the amount of debt exceeds the FMV of the subdivision, X would also realize income from the discharge of indebtedness.

(i)

Inclusion of Purchase-Money Mortgage in Basis: It is a basic rule that the cost basis of purchased property should include the amount of a mortgage. There are however two exceptions: (1) A purchase-money mortgage so greatly in excess of the propertys value that the buyer is unlikely to pay it off is unlikely to be included in basis. (2) contingent and indefinite obligations of the buyer are not included in the propertys depreciable basis. Negative Basis: The IRS has a long standing position that property cannot have a negative basis. Thus a taxpayers basis could not be reduced below zero by percentage depletion deductions that exceeded the propertys basis. Cancellation-of-indebtedness aspects of the Crane Rule: In the following hypotheticals, focus on whether there is a disposition of property, and realization of gain (from basis) from the debt being forgiven. Just b/c cash is not being given does not mean that T cant realize income (gain)

(ii)

(iii)

57

**HYPO: Debtor transfers property with a basis of $5k, FMV of $6k to a creditor in full satisfaction of a $7500 debt, which was secured by the property. RESULT: If the debtor was personally liable on the indebtedness, he realizes $1000 gain on the disposition of the property and, under T.R. 1.1001-2(c), $1500 of discharge of indebtedness income. If not personally liable, Tufts treats the full amount of indebtedness as an amount realized on the transfer of the property to the lender, i.e., $2500 of gain. HYPO: Same facts but the debtor persuades the lender to accept $5k cash in full discharge of the $7500 indebtedness. (Since the encumbered property is not disposed of, there is no occasion to compute any amount realized) RESULT: $2500 of income via discharge-of-indebtedness. **HYPO: Debtor pays a lender $5000 cash in full discharge of $7500 of qualified real property business indebtedness (See 108(a)(1)(D)). The real property had FMV of $6k, and had a basis of $5k. RESULT: Of the $2500 discharged, the borrower may elect to exclude $1500 from income and apply that amount against the basis of his property (See 108(c)), thus reducing the basis to $3500.

(iv)

Non-purchase-money mortgages: HYPO: T buys property for $50k, properly deducting $30k of depreciation thereon, and then borrows $40k from bank on a non-recourse note, using property as security. RESULT: Eventhough the loan proceeds exceed the propertys basis, no gain is recognized, and thus the basis of the property is not increased *See Woodsam C. Tax Shelters 1. How Tax Shelters Work 2. The At-Risk Rules 3. The Passive-Loss Rules D. The Basis of Property Acquired by Gift 1. In General: As we saw in Chapter 3, the recipient of a gift can

exclude the gift from income under 102(a). A donor, who transfers the property by gift usually recognizes no gain or loss on the transfer. Per 1015(a), the donee generally assumes the donors basis for the property (carry-over). Thus this may result in taxing ht donee on gain attributable to appreciation that occurred while the donor held the property. Lower-of-basis-or-value: If at the time of the gift the value of the property is less than the donors adjusted basis (basis is greater than FMV), the donee must use the FMV (rather than the donors basis) in computing the amount of loss realized on a disposition of the property. 1223(2): If you get property, you must carry over basis and use holding period of the donor, i.e., if Donor had property for 6mo., and gave it to Donee, donee has holding period of 6mo. See problems/handout page 539 Farid-Es-Sultaneh v. Commn

58

F: 1) In 1924, in contemplation of marriage, Kresge transfer stock as a conditional gift to FES. 2) Shortly thereafter, FES gives valid consideration for the gift, as she relinquished all of her dower and other marital rights including the right of support. 3) They get married, and divorce 4 yrs. later. In 1938, FES sells some of the stock at $19/per share. Basis of Kresge was around 15 cents per/share. H: This was not a gift (although it was labeled as such), thus T does not acquire Kresges basis. This was a sale as the stock was transferred in consideration of Ts promise to relinquish marital rights. When she acquired the stock, FMV was $10this is her basis. 2. Marriage/Divorce Transfers (i) Davis case and 1041: US v. Davis: T transferred appreciated stock to his former wife under a divorce property-settlement agreement. The court required T to recognize the gain to the extent that he value of the stock exceeded its basis. The Ts ex-wife obtained a cost basis equal to the value of the stock received. In essence, the transaction was viewed as if the transferor (husband) had paid cash to his ex-wife, who had then used the cash to purchase the property. 1041: In overturning Davis, 1041 instructs that gain or loss is not recognized upon the transfer of property to the taxpayers spouse or, in the case of transfers incident to divorce, to a former spouse. -The exclusion for transfers incident to divorce covers all transfers w/in one year of the cessation of the marriage, as well as transfers related to that cessation. -1041 treats the property (for income-tax purposes) as having been acquired by the transferee by gift. -Although a gift, the basis of such property is not governed by 1015(a), but rather 1041(b)(2). See 1015(e), which instructs marital property transfers to 1041. -Under 1041(b)(2), the transferee assumes the transferors basisthe only practical difference is that this does not contain the lower-ofbasis-or value limitation, which is discussed above. See Problems/Handout page 545 E. The Basis of Property Acquired by Bequest or Inheritance 1. Present Law: 1014the basis of inherited property is usually the value of the property at the date of the decedents death. Since the decedent is not required to recognize gain on the transfer of the property at death, any appreciation that occurred during the decedents life is exempted from tax. (Stepped-Up basis) See Problems page 556 HYPO: Basis in stock $100, FMV $10,000. Ms brother in law is ill, gives him stock. Brother in law takes basis of $100 (gift). B-I-L, then bequests the stock back to M, who will get the stepped up basis. RESULT: 1040(e) prevents such bush-league tactics. HYPO: $100 basis stock now has a FMV of $1M. Huge gain is up, how can you avoid? Go to bank and borrow $990,000 (as 1M stock as security). You get this money tax free, as it is borrowed money. If T dies in an accident. His heirs get the 1M (stepped-up) basis, and get a deduction for the 990k loan, based upon the Fed. Estate Tax. 2. Law after Estate-Tax Repeal: 1022 3. Income in Respect to a Decedent See problem on page 564.

59

Chapter 10 CAPITAL GAINS AND LOSSES A. The Significance of Characterization Whenever a gain is recognized, it must be characterized as ordinary or capital Capital Gains must be fully included in gross income. Long-term capital gains are generally taxed at a lower rate than ordinary gain. Capital losses are generally less attractive than ordinary losses 1. Capital Gains/Losses: 1222 defines capital gains/losses as gains and losses from the (a) sale or exchange of, (b) capital assets. (both must be present) The term capital asset is defined in 1221, means property held by the taxpayer (whether or not connected with his trade or business, other than that excluded by 1221 (a) (1) inventory (2) depreciable or real property used in trade or business (3) certain copyrights and artistic compositions (4) accounts or notes receivable from inventory or in the ordinary course of trade or business. Some classic examples of capital assets are: *stocks, bonds, securities held by a taxpayer (in his/her personal account) who is not a professional dealer in stocks/securities *undeveloped land held as an investment by the T *paintings or other artifactsgold, silver, stamps, coins, gems, etcprovided that they are not held for sale by a dealer *Ts home (note that if the home were sold at a loss, a deduction for the loss would not be allowed, thus the question of capital loss treatment would not arise. Any gain on the sale of the home (beyond the allowed exemption) would be a capital gain) *Household furnishings *A car used for pleasure or commuting *Investment property Some classic examples of non-capital assets are: *property held mainly for sale to customers or property that will physically become a part of the merchandise that is for sale to customers *Supplies regularly used in ordinary course of trade or business 2. Long Term/Short Term: Depends upon the Ts holding period. The holding period is significant b/c only long-term gains get the benefit of the capital gains preference. A gain or loss is long-term if the asset was held for more than one year before being sold/exchanged; 1222(3)(4) 3. Mechanics of Gain/Loss: If T has both capital gain and loss, the gains are not netted against the losses in computing gross income; instead the losses are deducted in computing AGI; 62(a)(3) Net Capital Gain (NCG) is subjected to the favorable capital gains tax rates. NCG is computed by taking NLTCG and subtracting NSTCL. -If the LTCG exceeds the LTCL (the excess is NLTCG), one next nets all the short term transactions. Any excess of STCL over STCG (that is, any NSTCL) is then subtracted from the NLTCG to arrive at the NCG. HYPO: STCG = $5,000

60

STCL = LTCG = LTCL =

$7,000 $14,000 $4,000

NSTCL: 2,000 NLTCG: 10,000 NCG: 8,000 (This is what is taxed favorably) Any excess of short-term capital gain (NSTCG) over NLTCL is taxed at ordinary income tax rates. HYPO: LTCG = LTCL STCG STCL $10,000 = $5,000 = $10,000 = $12,000

NLTCG: $5,000 NSTCL: $(2,000) NCG: $3,000 -If NSTCL was ($10,000), it would wipe out all of the NLTCG (5k), and still leave a NLTCL of $5,000. -Thus, 3,000 would be deductible and 2,000 carried forward as a STCL to offset capital gains of later years. ***From the Ts point of view, the first prize is to obtain a net capital gain for the year. The consolidation prize is a capital loss that can be used to offset up to 3,000 of ordinary income. The 3,000 offset only occurs if there is a deductible capital loss left over after offsetting capital gains. 4. Capital Losses: Losses from sales or exchanges of capital assets shall be allowed only to the extent allowed in 1211 and 1212 a) 1211: Corporate taxpayers may deduct capital losses only to the extent of capital gains, with excess capital losses carried over to other years. b) 1212: 5. Policy Considerations In Taxation of Capital Gains: Several reasons have been offered in support of taxing capital gains at lower rates than those on ordinary income: (a) Limit the tax that results from the bunching in a single year of gains accrued over many years HYPO: T of modest income owns stock, which has gradually appreciated in value over the past twenty years. If the T sells the stock, 20 yrs. appreciation may be taxed in a single year. (b) Inflation Rationale: Such gains may reflect inflation rather than real changes in economic value (c) Capital-Formation Rationale: People will increase their investment in assets that produce capital gains (prevent locking-in) B. The Sale-or-Exchange Requirement

61

1222 limits capital gains and losses to those from the sale or exchange of capital assets. (compare 1001(a)sale or other disposition of property) Thus some courts have held that some taxable dispositions are not sales or exchanges and therefore do not result in capital gain or loss even if the property disposed of is a capital asset. Nahey v. Commn Company acquired the Wehr Corp., whom had a pending claim against Xerox. Company acquires right to lawsuit and ends up getting a 6M settlement from Xerox. H: Not a capital gain, as the settlement of the lawsuit b/t the companies does not constitute a sale or exchange. The receipt of payment by a creditor is not traditionally viewed as a sale or exchange b/c the obligation was extinguished by the payment (i) Transfer of Property in Satisfaction of debtors obligation: HYPO: D owes C $100. D transfer to C a capital asset with a basis of $40 and a value of $100. Ds recognized $60 (gain) is in fact arisen from a sale or exchange. (ii) Statutory Sales and Exchanges: Code provides that transactions which normally would not be sales or exchanges are to be treated as sales or exchanges. -331: amount received by a shareholder in liquidation of a corp. -1271(a): amount received on retirement of debt instrument -165(g): losses on worthless securities -166(d): loss on business bad debt

C. Capital Assets: Business and Purpose under 1221(a)(1): This excludes inventory from the definition capital asset. (Property that is held by the T primarily for sale to customers in the ordinary course of his trade or business.) The issue becomes then, does the T have a trade or business? The Ts whose activities are so frequent or intensive that he is held to be in business is said to be a dealer in the property sold. If he is not a dealer, gains and losses will be capital. Kemon v. Commn I: Were securities are held primarily for sale to customers in the ordinary course of business? H: No. R: In determining whether a seller of securities sells primarily to customers, the seller must be comparable to a merchant, in that they purchase their stock in trade (here securities) with the expectation of reselling at a profit, not b/c of a rise in value, but merely b/c they hope to find a market of buyers who will purchase from them at this price. This is a dealer. Contrast that with a tradersellers depending upon a rise in value or an advantageous purchase to enable them to sell at a price in excess of cost. Here, the activity of T with regard to the securities conformed to the customary activity of a trader rather than a dealer holding securities primarily for sale to customers. T frequently refused to sell securities which it was accumulating despite the fact that the bid price would have resulted in profit. T never advertised itself or held itself out to the public as having on hand securities for sale. A trader is someone who buys and sells securities on a large scale but whose gains and losses are capital b/c the sales are not to customers A Letter ruling listed a number of factors to be used in identifying dealer property: (1) Purpose for which the property was initially acquired; (2) Purpose for which the property was subsequently held; (3) Extent to which improvements, if any, were made to the property by the T;

1.

62

(4) The frequency, number and continuity of sales; (5) Extent of advertising, promotion, or other active efforts used in soliciting buyers for the sale of the property (6) Purpose for which the property was held at the time of sale. 2. Non-Capital Assets Other Than Inventory 1221(a)(2): Depreciable personal as well as real property used in a trade or business. The two critical definitions relating to this category is that the property must be usedas opposed to lying idle, and the use must be in a trade or businessas opposed to an investment activity or a personal activity. However, although not classified as a capital asset, he sale or exchange of this property may give rise to capital gain or lossSubject to 1231; the other below this line are not. ------------------------------------------------------------------------------------------1221(a)(3): Copyright, literary, musical or artistic compositionThis reflects the notion that gains from labor are not capital gains 1221(a)(4): Deals w/ a problem faced by accrual-method Ts who sold goods/services for notes (or accounts receivable) took the amount of the notes into income, and subsequently realized losses on the sale of the notes. 1221(a)(5): 1221(a)(7): Excludes hedging transactions from capital asset treatment. Hedging transactions generally used by a firm to manage the risk of price changes with respect to its ordinary (inventory) property. -i.e., a purchase of stock in order ensure a source of raw materials. 1221(a)(8): Ordinary business supplies are not capital assets. D. Quasi-Capital Gains and Losses Under 1231: 1231 determines the character of gain/loss on three kinds of taxable dispositions of property: (a) Sales or exchanges of 1231 assets that is property used in the trade or business, which is excluded from the status as a capital asset per 1221(a)(2). (b) Condemnations of 1231 assets and long-term capital assets held in connection w/ business or investment activity. (c) Involuntary conversions by destruction or theft of 1231 assets and long-term capital assets held in connection w/ business or investment activity.(casualty transactions) 1. Process of 1231: (1) The Casualty Fire Pot: The gains and losses on all casualty transactions are netted together. If that net result is a loss, then none of these individual casualty transaction enter further into the workings of 1231. If the net result on all the casualties is to break even or is a gain, then all of the casualtiesgains and lossesinvolved in this netting process pass into consideration in the Main Pot (2) The Main Pot: The following transactions are netted in the Main Pot (a) All casualties, if they have netted to break even or a gain in the Casualty Pot (b) Sales or exchanges of capital assets or depreciable property and real property used in the trade or business, held for more than one year (c) Condemnation of depreciable property and real property used in the trade/business, held for more than one year. If the transactions in the Main Pot net out to a gain, then each transaction in the Main Pot will be considered long-term capital gain. These 1231 (quasi) capital gains/losses are then combined

63

with the taxpayers other capital gains and losses arising from a sale or exchange of capital assets. The combined gains/losses are subject to the normal treatment If all the transactions in the Main Pot net out to break even or a loss, then each transaction in the Main Pot is considered an ordinary gain or loss. See Problems/handout, page 633 2. Background of 1231 and 1221(a)(2): Until 1938, land, buildings and machinery were capital assets except when held for sale by a dealer. Thus gains and losses on the sale were capital gains/losses. However, capital loss treatment discouraged sales of depreciable business property, since the T could get ordinary deductions by keeping the property and taking depreciation deductions. HYPO: T had trouble selling his house, so he rented the property for four years, claimed depreciation and ultimately sold the property for a loss. RESULT: The property is not a capital asset, but an asset described in 1221(a)(2), 1231, b/c Ts actions (renting, taking depreciation) established that the property was used in a trade or business. E. Depreciation Recapture and Related Matters: The basic theory of recapture is that the gain on the sale of depreciable property which is attributable to previously taken depreciation is deemed to be ordinary gain. 1. Sections 1245 and 1250 (a) 1245 Gain from dispositions of certain depreciable property: Overrides 1222 and 1231 to recapture as ordinary income the make-up gain (does not apply to losses) on 1245 property which is defined as any property which is or has been property of a character subject to the allowance for depreciation provided in 167, and is either (i) personal property (ii) other property *This is said to apply to personal property such as machinery, equipment, vehicles HYPO: T pays 10k for a truck, properly deducts 6k in depreciation while using the truck in a business, and then sells the truck for $5500. The Ts gain on the sale is $1500 ($5500 less $4000 adjusted basis). This $1500 is not a real economic gain, but a make-up gain that just compensates for the excess depreciation deductions taken. (An economic gain results where property is sold for more than its cost.) 1245 treats this as ordinary gain. HYPO: T pays 100k for a machine, and depreciates for 6 years (10k per, 60k total) leaving basis of 40k. T then sells the machine for 130k. RESULT: T realizes and recognizes $90k. Under 1245, 60k is ordinary income (not involved in the netting process of 1231) b/c of the 90k gain, 60k was due to previously taken depreciation. 30k is put into the Main Pot where it may turn out to be ordinary income or capital gain in the 20% group. HYPO: Jones buys a machine for his trade or business for 16k. J took depreciation deductions totaling 6k, rendering his basis 10k. J then sold the machine for 20k. RESULT: Realized gain is 10k. 6k is ordinary income, 4k goes into the 1231 Main Pot HYPO: Same facts, except that the machine was sold for 12k. RESULT: 2k gain (of ordinary income). Nothing passes into the Main Pot. The only reason this gain came about was b/c of the depreciation deduction. (b) 1250 Gain from dispositions of certain depreciable property: This governs depreciation recapture on most buildings, and is easier on the T than 1245. If 1250 property has been held

64

for more than one year, only additional depreciationdepreciation deductions in excess of those that would have been allowed under straight-line depreciationis recaptured. Since 1986, real property must be depreciated on straight line basis, thus, this is only meaningful to sale of real estate pre-1986. 1250 property is any real property (other than 1245 property) which is or has been property of a character subject to the allowance for depreciation provided in 167 HYPO: T purchases an office bldg. for 100k in 1975 and it has a useful life of 30yrs. Straight line method depreciation in the first year would be $3,333, thus the double declining method would be $6,666. T, after choosing double-declining, sells the bldg for 100k after one year. His gain is the amount realized (100k), minus adjusted basis (93,334) or $6,666. Per 1250, the excess of the accelerated depreciation over the straight line is 3,333. RESULT: Thus on the sale, $3333 of the gain would be ordinary, and $3333 would go into the 1231 main pot. (Note that if the bldg. were a machine, 1245 would treat the entire $6,666 as ordinary income.) 2. Section 1239: SALE OF DEPRECIABLE PROPERTY TO A RELATED PARTY 1239 deals w/ the problem of capital gain followed by depreciation deductions taken by a taxpayer closely related to the taxpayer who realized the gain. (a) In the case of a sale or exchange of property, directly or indirectly, between related persons, any gain recognized to the transferor shall be treated as ordinary income if such property is, in the hands of the transferee, of a character which is subject to the allowance for depreciation provided in 167 (b) Related persons means (1) a person and all entities which are controlled entities with respect to such person (2) a taxpayer and any trust in which the T (or spouse) is a beneficiary, unless such beneficiarys interest in the trust is a remote contingent interest. (3) except in the case of a sale or exchange in satisfaction of a pecuniary bequest, an executor of an estate and a beneficiary of such estate. (c) Controlled entity means (1) person owns more than 50% of the value of the outstanding stock (2) person owns more than 50% of capital or profit interest in partnership HYPO: T owns depreciable property w/ a basis of 2k and a value of 10k, and sells/exchanges the property with his company (who will be taking depreciation after taking 10k basis). 1239 recaputres this additional depreciation before it is taken by making gains on the sale of depreciable property b/t related persons ordinary income. HYPO: H purchased depreciable property for 20k, property deducted 12k (basis 8k) and then sold it for 30k (FMV) to Y Corp. H owns all of the stock of Y Corp. RESULT: Hs gain 22k of ordinary income RESULT: If Y corp. was instead owned by Hs adult daughterthen. 3. Section 1231 Reconsidered: F. CAPITAL GAINS & THE CHARITABLE CONTRIBUTION DEDUCTION HYPO: 70%-bracket taxpayer owned a fully depreciated (zero-basis) airplane worth 100k. A sale of the airplane would have left the T w/ only 30k. A contribution of the plane would allow the T to notrecognize the gain of the property and also deduct the full value of the propertythus a double benefit. To combat this, 170(e) reduces the amount (somewhat) of charitable contribution deduction

65

-170(e)(1)(A)deductions are limited to the basis of ordinary-income and short-term capital gain property -Contributions of long-term capital assets and 1231 assets are still deductible in full, with the following two exceptions: (i) Tangible personal property if use by the donee is unrelated to the purpose of function constituting the basis of for its exemption (ii) to or for use of a private foundation CHAPTER 9 NON-RECOGNITION TRANSACTIONS AND (DIS) ALLOWANCE OF LOSSES *A realized gain is includable in gross income only to the extent that it is recognized. *Similarly, a realized loss cannot be deducted unless it is recognized. 1. Non-Recognition Rules: Congress had decided (for policy reasons) that immediate recognition of the realized gain or loss is inappropriate. These rules act to postpone the tax reckoning for certain types of transaction. -Non-recognition usually results in a deferral of the gain or loss rather than a permanent exclusion of the gain or loss from the tax base

A. Tax-Free Exchanges: One item of property is exchanged for qualifying non-recognition


property The basis of the property given up is usually transferred to the property acquired. If the taxpayer receives cash or non-qualifying property (boot) in the exchange, any realized gain usually must be recognized to the extent of the boot received.

(1) Like-Kind Exchanges Under 1031: 1031 generally provides for non-recognition
of gain or loss if property is held for productive use in a trade or business or for investment is exchanged solely for property of like-kind, to be held either for productive use in a trade or business or for investment. 1031 (a)(1): No gain/loss shall be recognized on the exchange of property held for productive use in a trade or business or for investment if such property is exchanged solely for property of like kind, which is to be held either for productive use in a trade or business or for investment. (a)(2)does not apply to stock, bonds, notes, interests in partnership, etc (b)Not Solely in Kind: if a like kind exchange takes place, along with other property being received, then the gain, if any, to the recipient shall be recognized, but in an amount not in excess of the sum of such money and the FMV of such other property. (d)Basis: If property was acquired on an exchange described in this , then the basis shall be the same as that of the property exchanged, decreased in the amount by any money received by the T and increased by the amount of gain or decreased in the amount of loss to the T that was recognized on such exchange. (i) Like-Kind Property: T.Reg. 1.1031(a)-1(b): Indicates that like-kind is a reference to the nature or character of the property and not to its grade or quality. One kind or class of property may not be exchanged for property of a different kind or class. -Livestock of different sexes are not like kind. 1031(e) -Foreign real property is not of like kind w/ real property located in US

66

-Personal property used predominantly in US is not of like kind w/ personal property used outside US Depreciation of like-kind exchanges: -The property received is to be depreciated over the remaining recovery period of the property exchanged using the same convention and depreciation method as was used for the property exchanged. -However, if the new basis exceeds the old, any increase of the new propertys basis (over the old) is treated as a newly purchased property. HYPO: T exchanges truck (basis 20k) and 15k cash for a new truck. The basis of the new truck would be 35k (20k basis of old truck plus 15k additional cash invested). As to the 20k, T would compute depreciation using the method, convention and remaining recovery period of the old truck. The remaining 15k would be depreciated as if a new truck had been purchased. (ii)

B. Tax-Free Rollovers: Property is old and qualifying replacement property is purchased


by the T within some prescribed period of time.

67

Das könnte Ihnen auch gefallen